Jump to content

Wikipedia:Reference desk/Science: Difference between revisions

From Wikipedia, the free encyclopedia
Content deleted Content added
Line 483: Line 483:


[[User:Dauto|Dauto]] ([[User talk:Dauto|talk]]) 19:00, 20 November 2012 (UTC)
[[User:Dauto|Dauto]] ([[User talk:Dauto|talk]]) 19:00, 20 November 2012 (UTC)

== Recycling 10, 20, or 30 year old paper ==

Hello, I have some old documents in boxes, and these papers are probably 10, 20, or 30 years old. I intend to recycle all of these documents, and I was wondering, can they even reuse the fibers from paper that old? [[Special:Contributions/2A02:AF8:1:3500:0:0:0:9868|2A02:AF8:1:3500:0:0:0:9868]] ([[User talk:2A02:AF8:1:3500:0:0:0:9868|talk]]) 21:30, 20 November 2012 (UTC)

Revision as of 21:30, 20 November 2012

Welcome to the science section
of the Wikipedia reference desk.
Select a section:
Want a faster answer?

Main page: Help searching Wikipedia

   

How can I get my question answered?

  • Select the section of the desk that best fits the general topic of your question (see the navigation column to the right).
  • Post your question to only one section, providing a short header that gives the topic of your question.
  • Type '~~~~' (that is, four tilde characters) at the end – this signs and dates your contribution so we know who wrote what and when.
  • Don't post personal contact information – it will be removed. Any answers will be provided here.
  • Please be as specific as possible, and include all relevant context – the usefulness of answers may depend on the context.
  • Note:
    • We don't answer (and may remove) questions that require medical diagnosis or legal advice.
    • We don't answer requests for opinions, predictions or debate.
    • We don't do your homework for you, though we'll help you past the stuck point.
    • We don't conduct original research or provide a free source of ideas, but we'll help you find information you need.



How do I answer a question?

Main page: Wikipedia:Reference desk/Guidelines

  • The best answers address the question directly, and back up facts with wikilinks and links to sources. Do not edit others' comments and do not give any medical or legal advice.
See also:


November 14

Observation in quantum mechanics

Since wave function collapse requires someone to observe the system, does this mean that before humans (or other life), the Universe didn't exist? --168.7.231.3 (talk) 02:05, 14 November 2012 (UTC)[reply]

"Observation" is just the word we use for the event that occurs when a wavefunction collapses. It simply means that a certain kind of physical interaction has occurred. It does not require that any living thing be present. The confusion is understandable, but wavefunction collapse can happen whether or not humans are looking at the results. In fact, even physicists have a hard time defining what interactions will or will not constitute measurement. See measurement in quantum mechanics, wavefunction collapse and quantum decoherence. Dragons flight (talk) 03:26, 14 November 2012 (UTC)[reply]
Right. Just to be redundant, tyhis relies on the principle that there is no action at a distance, and that observation means somebody's bounced a photon off it or something similar. Which naturally means that the wavefunction is no longer what it was before you observed it. Gzuckier (talk) 01:44, 16 November 2012 (UTC)[reply]

I think I've found an error that make the information in the article contradicting. In the "even perfect numbers" section, it says: "As of June 2010, 47 Mersenne primes and therefore 47 even perfect numbers are known." Then later in the section it says: "It has not yet been proved that there are (or are not) others after the 41st." So I think it should have said "...other after the 47th."?174.20.101.190 (talk) 06:17, 14 November 2012 (UTC)[reply]

This is science reference desk, you should have asked the same on mathematics reference desk. Sunny Singh (DAV) (talk) 07:48, 14 November 2012 (UTC)[reply]
The explanation is quite simple. There are 47 known Mersenne primes, but only the first 41 are known to be consecutive - not all possibilities after the 41st Mersenne prime have been checked, so there may be as yet undiscovered Mersenne primes in the gaps between the 41st and the 47th on the known list. There is a 1-1 correspondence between Mersenne primes and even perfect numbers, so there are 47 known even perfect numbers but only the first 41 are known to be consecutive. This is what the perfect number article is trying to say - but I agree it is not very clear, and the wording could be improved. Gandalf61 (talk) 10:41, 14 November 2012 (UTC)[reply]
Thanks, Gandalf61. I've clarified it there. Duoduoduo (talk) 14:53, 14 November 2012 (UTC)[reply]

Heat energy vs thermal energy

Is correct to say that heat energy and thermal energy are different ? Sunny Singh (DAV) (talk) 07:36, 14 November 2012 (UTC)[reply]

There is a difference in a subtle way. Note that the term heat energy is bad english - heat is energy, so the term is like saying energy energy. The terms heat and thermal energy are often used interchangeably. However, more correctly in a scientific or engineering paper, heat is energy being interchanged from one media to another (e.g, the heat rejected by a gas to its surroundings when it is compressed), whereas thermal energy is the thermodynamic energy in a system. You may like to carefully read the WP articles Heat and Thermal Energy. Wickwack 58.169.249.183 (talk) 09:00, 14 November 2012 (UTC)[reply]
I was taught that heat was another term for waste energy. Plasmic Physics (talk) 11:46, 14 November 2012 (UTC)[reply]
In most engineering applications, heat is waste energy, i.e., unwanted or unusable, but in some cases it is the heat that is wanted. For example, the heat rejected in an internal combustion engine is waste, unless it is utilised in co-generation. The heat rejected by the burning fuel in a power station is not waste heat, but what's left over after boiling the feedwater and has to be lost in the cooling towers is waste. You may have mis-remembered or misunderstood your teacher. Wickwack 120.145.170.99 (talk) 15:00, 14 November 2012 (UTC)[reply]
No, I didn't mis-remember or misunderstand my lecturer, he used the second law of thermodynamics to explain it. Plasmic Physics (talk) 22:30, 14 November 2012 (UTC)[reply]
The 2nd law is essentially that in the conversion of heat into mechanical work, not all the heat can be so converted. How does that mean that that heat is waste energy? Not only is not all the heat "wasted", heat may be the desirable output as I said. Wickwack 120.145.143.165 (talk) 00:48, 15 November 2012 (UTC)[reply]
He didn't use "heat" in that sense. Plasmic Physics (talk) 05:04, 15 November 2012 (UTC)[reply]
It seems odd to read that heat is waste. Although most of the heat produced throughout the world become waste, yet it is a useful form of energy. It is true that we don't notice most of the heat produced, but it is not like that heat is waste. I don't think so because heat is used in steam engines to evaporate water, in ovens, in heating effect of many devices and many more. Talking about the second law of thermodynamics, it says all the heat cannot be converted into mechanical energy but some can. Sunny Singh (DAV) (talk) 09:16, 15 November 2012 (UTC)[reply]
It isn't that "heat is waste", which is a rather trite saying and like all trite saying has a tiny bit of truth packed inside too much approximation to have any real meaning. We should avoid triteness when describing things scientifically. Heat is waste in the sense that when two substances are in contact they exchange heat: the warm one heats up the cold one, until such time as the temperature between the two is equal. Here's the thing: The energy transferred from the hot body to the cold body cannot be utilized again to do work. That's because commensurate with the energy transfer is an increase in entropy, and you cannot recover that energy without "stealing" entropy from somewhere else in the universe, which would require you to heat up that part of the universe by more than you recover. That's the inescapable part of the second law of thermodynamics. Now, you can use heat to do work, as it is in the process of moving from the hot place to the cold place, but once the two places are in thermal equilibrium, you can't use that energy to do work again in an absolute sense: that energy is lost to the universe, i.e. there is a loss of free energy. So, in one sense, the trite statement "heat is waste" is correct, however it is not wholly correct because you can do work with heat; you just can't recover the energy after the work is done. --Jayron32 14:02, 15 November 2012 (UTC)[reply]
First, that is off the mark. Take a red hot body and a cold body, say a large fired up rod and a cup of cold water. Let's put these together. Energy flows from one to the other, and yes, that energy can still do work, since it will create steam that can be used in a heat engine. As for other misconceptions regarding the second law, I intend to knock it out of its ivory tower too. --Modocc (talk) 14:22, 15 November 2012 (UTC)[reply]
Um, why do you start a response with, "that is off the mark" and then say something that agrees with everything I say 100%. I am confused as to what parts of my explanation are incorrect. --Jayron32 14:33, 15 November 2012 (UTC)[reply]
To clarify, you said this, "The energy transferred from the hot body to the cold body cannot be utilized again to do work." Since you said "cannot be utilized again" I assumed you understood that the hot body already did work on the cold body, by heating it up, with the kinetic energy of the molecules being transferred. Its incorrect to conclude that this energy that was transferred cannot do any work again as you said though. In any case, entropy itself is elusive in the sense that it is inherently system dependent. In the 70's, entropy and Gibbs free energy were used to explain why diamonds, at the time, could be created only under extreme pressure and temperature, but in the 80's new catalysts circumvented the presumed entropy. -Modocc (talk) 15:01, 15 November 2012 (UTC)[reply]
Really? Once the rod and the water are the same temperature, how do you presume to use the thermal energy that moved from the rod into the water to do work again? Energy which is transfered from one form to another, or from one location to another, can be used to do work indefinitely except energy which is transferred as heat. You can only use that energy once; as it makes its trip from the hot stuff to the cold stuff you use that transfer to do work; but only that one time. Other energy transfers can be used indefinitely to do work along the way. That's the difference between the first law and second law of thermodynamics. --Jayron32 18:12, 15 November 2012 (UTC)[reply]
Let the temperature of the rod be 200celius. Let the rod cool to 199degrees and the water heat up to 199degrees. Assuming my cup has a lid on it, the water has turned to steam and is in equilibrium with the rod. Now, some of the heat of that rod has done work and you have said its not able to do work again, but it can. Open the valve on the lid, so the steam enters an engine. The heat energy of this steam gets transferred to the engine such that it is again does work. -Modocc (talk) 18:45, 15 November 2012 (UTC)[reply]
When you open the cup, in order for the steam to do work, it needs to be opened into a region of a different temperature/pressure than the inside of the cup. When you do that, you introduce a new heat transfer (from the hot inside to the cold outside). So you're dealing with different heat. You no longer have a closed system of just the rod and water, and if you take your system to be cup+rod+steam engine, the heat transfers have not been complete until all three are in thermal equilibrium. Once all three are, your system loses its ability to do work. My point still stands as 100% correct. --Jayron32 05:41, 16 November 2012 (UTC)[reply]
Once the steam leaves the cup, and I shut the valve, the rod should not rapidly lose much more heat if the rod and cup are insulated well. But I'm not sure what you mean by "dealing with different heat." I do mean a new heat transfer, but this basically just means that this heat gets utilized again. Also, your choice of closed/open systems are arbitrary. Its my understanding that work performed on any object is equivalent to its change in kinetic energy. Let the water initially begin with an ambient temperature and the water is heated by the rod, the kinetic energy of the water molecules are increased (and energy is absorbed too in order to break bonds with its heat of vaporization). Then, when this steam is used in the engine, the water will lose this extra energy, with some of it doing work on the engine's pistons, but the rest will not, with these amounts depending on the engine's efficiency. But all of that extra energy over and above the water's ambient heat content which the engine receives, but which it does not fully utilize, was provided initially by the rod's heat doing work on the water to raise its temperature above the ambient temperature. Thus, the water's initial ambient heat content plus the transferred heat energy that does work on the water prior to opening the valve equals the sum of the engine's waste heat energy and the energy of the work done on its pistons. A portion of this rod's heat energy does work on the water, and then as a part of the water's heat energy, it either gets wasted or performs work again on the piston. Your statement that "The energy transferred from the hot body to the cold body cannot be utilized again to do work." is wrong. -Modocc (talk) 06:42, 16 November 2012 (UTC)[reply]
Once you've redefined the system, it can. The difference is that you're working from a different defined system each time. You can keep passing heat energy from one place to another until the heat death of the universe and keep doing work at each transfer. My statement never claimed you couldn't do that. What you cannot do is use heat energy twice within a closed system. --Jayron32 18:53, 16 November 2012 (UTC)[reply]
Err, this assertion regarding what can or cannot happen within closed systems is wrong too, but I'm done with this topic for now. -Modocc (talk) 01:50, 17 November 2012 (UTC)[reply]
Seems to me this discussion has lost the plot with all this talk of triteness, reusability, and diamonds. Seems to me Jayron has merely established that heat can be waste (and in any thermodynamic process, some will be waste in a component process of a larger process), but heat isn't necessarily waste. Maybe that is a trite (= not novel) thing to say, but it is clearly different to saying heat IS waste (always), which is what PlasmicPhysics said. Plasmic said something wrong. Wickwack 120.145.0.141 (talk) 16:06, 15 November 2012 (UTC)[reply]
Incidentally, in Engineering Thermodynamics, D B Splading and E H Cole, Edward Arnold publ, a standard undergraduate text for many years, in Chaper 5 Heat, it gives a formal definition of heat: "the interaction between systems which occurs by virtue of their temperature difference when they communicate" (page 86 in 3rd Edition). It then goes on with 7 more pages about what heat is and what it is not. There is not a word about waste or any similar word like it. I have several other textbooks on thermodynamics and they all give the same definition, albiet in their own words, without any statement about "waste" until they come to actual examples, where for example, in a Carnot process some of the heat must be unusable (wasted). Wickwack 120.145.0.141 (talk) 16:06, 15 November 2012 (UTC)[reply]
All too true, and the above usage of "waste" is simply being used synonymously for the concept of entropy. So to define entropy is to define waste. In my neighborhood, we do a great deal of recycling though, so as to reduce the amount of waste that gets wasted. We've been binging on fossil fuels for some time now, but its time to sober up and do a far better job of recycling energy than we have. -Modocc (talk) 16:31, 15 November 2012 (UTC)[reply]
I also said the same thing heat is not waste. According to me, the overall discussion goes in favor of Modocc and Wickwack. Sunny Singh (DAV) (talk) 12:08, 16 November 2012 (UTC)[reply]
Calling heat "waste" is kind of silly when you think that the Sun, the furnace in your house etc. are all providing "waste". Best to call heat heat and leave the rest to semanticists. As I understand it, given that it appears the universe never ceases to expand and thus to cool, the heat should never become unrecoverable to work at any time in the future; there is no heat death of the universe, though the article seems a bit unsatisfying on the point. Wnt (talk) 20:59, 17 November 2012 (UTC)[reply]
The fundamental definition of heat as given in modern textbooks is energy transfer from one body to another that is not due to macroscopic work. This definition does not involve thermodynamic concepts, because you need to have this definiton to build up the thermodynamic concepts. Macroscopic work is defined as energy transfer due to a change in the external parameters of a system. What one chooses as the external parameters (e.g. the volume) is entirely arbitrary in priciple, so this makes the separation of energy transfer into a work part and a heat part entirely subjective. If you choose to describe the system exactly in terms of all its fundamental degrees of freedom, then all of the energy transfers will be work, and the entropy will be the so-called fine grained entropy which is always equal to zero (which expresses the fact that there is no loss of information at the fundamental level). Count Iblis (talk) 19:17, 16 November 2012 (UTC)[reply]

New Sub-Question:-
Leave this long discussion as it doesn't answers my question. Talk about the question. I read the article heat and thermal energy for another time as suggested above, but I have some confusions. I'll thankful if someone happily resolves my confusions.

  1. When two bodies having different temperature are brought in contact with each other, both bodies attain thermal equilibrium after sometime. Here during energy transfer, heat is transferred between bodies or thermal energy i.e., which one of the latter two is being transfered. Sunny Singh (DAV) (talk) 05:45, 18 November 2012 (UTC)[reply]
  2. Section 'Distinction of thermal energy and heat' of the article thermal energy mentions "Statistically, thermal energy is always exchanged between systems, even when the temperatures on both sides is the same, i.e. the systems are in thermal equilibrium. However, at equilibrium, the net exchange of thermal energy is zero, and therefore there is no heat". In the first sentence it says thermal energy can be exchanged when the systems are at thermal equilibrium and in the second sentence it says thermal energy cannot be exchanged. How is it possible ? Sunny Singh (DAV) (talk) 06:21, 18 November 2012 (UTC)[reply]
Answer to #1. Your english is a bit cryptic, so I hope I'm answering what I think you must be trying to ask. The correct term for what gets transfered is Heat. The reason why heat is the correct scientific term is bound up in the theory of thermodynamics. However, as I said in my first post, you will often find the terms heat and thermal energy used interchangeably in the literature. Thermal energy can mean other things, but whether an author means heat or something else can be determined by the context. The heat (which is a form of energy) gets transfered by one or more of the following ways, depending on just wht the bodies in contact consist of: Conduction (thru inter-molecular forces and collisions, and Radiation (electromagnetic radiation eg infra-red). If one of the bodies is a gas, then heat can be transfered within it by convection as well, sometimes referred to as mass transfer.
Answer to #2. What the author is trying to say is this: There is a two-way flow of heat between bodies when both are at a finite temperature - for bodies A and B there is a flow of heat from A to B (which would decrease the temperature of A ind increase it for B), and also a flow from body B to A (thus increasing A temperature and decreasing B). If the two bodies are at the same temperature, the two heat flows are equal, and since they are in opposite directions their effects in altering the temperatures cancel. You are perhaps surprised that there are two flows, and not just a flow from the hotter to the colder. To see why there are two flows, consider two bodies separated by a gap. Because each body is at some temperature (ie not at absolute zero), it must be emitting radiation, which is a function only of its temperature and its own properties (area, emissivity), which propagates across the gap as electromagnetic energy. After a propagation delay, the radiation hits body B, which must absorb some of it, as a function of its transparency. Meanwhile, Body B must be also radiating, since radiation is a function only of its own temperature and properties. If the two bodies are touching, then heat can flow by conduction (and perhaps convection) - that really doesn't change the fact that as both bodies are at some non-zero temperature, both must be emitters.
I am sorry that the discussion went away from just answering your original question, and I'm partly responsible of causing that when I corrected the erroneous assignment of heat as "waste" by PlasmicPhysics. Why he brought in the term "waste", which is not generally used in thermodynamic textbooks, is something only Plasmic can know.
Wickwack 121.215.50.212 (talk) 10:15, 18 November 2012 (UTC)[reply]

Two Last Sub-Questions:-

  1. Suppose I have two iron block of 1kg. I, somehow, increased the volume (mass remaining the same) of first block and increased the mass (volume remaining the same) of the second block. Assuming all blocks have the same temperature, which one has more thermal energy ?
  2. Suppose I have a solid block of iron of 1kg and gaseous nitrogen in a large container, nitrogen has also the same mass of 1kg. Which one - iron block or gaseous nitrogen- has more thermal energy ? Sunny Singh (DAV) (talk) 03:59, 19 November 2012 (UTC)[reply]
It would be better to ask new questions as new questions, not as sub-questions to an old question. This is because (a) you are more likely to get answers from various contributors, and (b) given the length of discusion on this one, probably nobody wants to contribute any more, even if they are still looking.
Answer #1: This question cannot sensibly be answered, as you cannot in any real way take a block of iron and alter its mass or its volume without changing both in proportion. By increasing the temperature, you can increase the volume a tiny bit without changing the mass, but the tiny change possible is not relavent to thermodynamics, and you excluded a change in temperature anyway.
Answer #2: Each element and each chemical compound has a specific heat that can in theory be deduced from its molecular and electron orbit structure. Specific heat is the amount of heat that a unit mass of a substance absorbs when its' temperature is increased by 1 unit (1 kelvin, 1 degree farenheit, etc). Particularly for gasses, specific heat varies with temperature. From standard tables, iron at 25 C (ie solid iron) has a specific heat of 450 J / kg.K; nitrogen has a specific heat at constant volume of 742 J / kg.K. Nitrogen, weight for weight, holds the greater amount of energy.
When talking about heat contained within a mass, it is necessary to know what this heat could be transfered to. The above answer is relavent if any heat flow in or out of the 1 kg iron or nitrogen is with respect to something else at close to 25 C. If the nitrogen was coolled so that it liquifies, for example, it would give up additional heat according to its latent heat of vaporisation.
Wickwack 58.170.164.182 (talk) 11:09, 19 November 2012 (UTC)[reply]

I was not talking about heat, I was talking about "thermal energy"; this is too late, so, close this discussion, I'll ask the same later. Thanks to those who contributed to this question and special thanks to Wickwack. Sunny Singh (DAV) (talk) 13:51, 19 November 2012 (UTC)[reply]

Vitamin D

Vitamin D is already present in cow's milk or it is added to milk by humans. Sunny Singh (DAV) (talk) 07:52, 14 November 2012 (UTC)[reply]

Most of the vitamin D in milk is artificially added. I can't find a source online that says exactly how much vitamin D is naturally present. Someguy1221 (talk) 08:20, 14 November 2012 (UTC)[reply]

Yes, I read the same thing in the last line of second paragraph of the article vitamin D. I am also confused how much vitamin D is naturally present in cow's milk. What about mother's milk (referring human), it contains vitamin D or not. Sunny Singh (DAV) (talk) 08:40, 14 November 2012 (UTC)[reply]

I found the same thing as with cows milk. It contains some vitamin D, but not enough to meet even a baby's daily requirements, but I didn't find a source that gave the exact amount. Someguy1221 (talk) 09:42, 14 November 2012 (UTC)[reply]
This site says that milk in the UK is not routinely fortified with Vitamin D. I also found this: "Whole cow’s milk was found to contain 38 i.u. vitamin D/I. Whole human milk contained 15 i.u. " from "The total content of vitamin D in cow's milk and human milk", Leerbeck (1980) - Journal of Human Nutrition. (There is a link on Google, but I've had a lot of trouble trying to get it to download and I'm not sure about putting the reference in here.) --TammyMoet (talk) 10:37, 14 November 2012 (UTC)[reply]

According to recent research, mother's milk contains enough vitamin D for the baby (more than 400 IU per liter) provided the mother's calcidiol level is above 120 nmol/l, which requires a daily intake for the mother of at least 4000 IU/day [1]. This is one of the arguments for higher calcidiol levels (120 nmol/l or higher instead of 50 nmol/l) despite there not being strong evidence for better health outcomes for adults, or that evidence being disputed. Obviously if babies would routinely need vitamin D pills because they would otherwise not even get the 400 IU/day for which we know their health is adversely affected, then that also implies that adults need more than 4000 IU/day. If we are right about this not being necessary for cancer or heart disease, then for other reasons we don't know anything about yet.Count Iblis (talk) 16:56, 14 November 2012 (UTC)[reply]

The use of ultraviolet light to enrich milk with vitamin D was discovered by Harry Steenbock. It's not so much that the vitamin D is added (there's is not some separate container of vitamin D that is mixed into the milk), rather the milk is vitamin D enriched/fortified, as there are pre-vitamin D compounds in milk that are transformed by the ultraviolet light into active vitamin D, in the same fashion sunshine (which contains ultraviolet light) can convert pre-vitamin D compounds to vitamin D in the skin. -- 205.175.124.30 (talk) 22:46, 14 November 2012 (UTC)[reply]

USDA's National Agricultural Library has all sorts of interesting information. This and this compare the nutrient content of 3.25% whole milk with and without added Vitamin D. Zoonoses (talk) 07:07, 15 November 2012 (UTC)[reply]

Forest fire vs candle flame

Candle flame goes off by the effect of wind but forest fire increases with increasing velocity of wind. Why ? Sunny Singh (DAV) (talk) 09:03, 14 November 2012 (UTC)[reply]

If you had a wind in proportion to the size of the flame, you could blow it out. For a forest fire, you'd need a hurricane, at least. StuRat (talk) 09:33, 14 November 2012 (UTC)[reply]
Also a forest fire is much, much hotter than a candle flame. Even if you could "blow out" the actual flames of a forest fire, the timber would still be burning. You can't really blow out any fire other than a tiny one such as a candle flame.--Shantavira|feed me 09:45, 14 November 2012 (UTC)[reply]
I disagree. You'd just need a long and powerful enough wind to allow for the heat to dissipate. This might be several hours at several thousands of miles per hour, though. StuRat (talk) 09:49, 14 November 2012 (UTC)[reply]
I think Shantavira was talking about the real world. Caesar's Daddy (talk) 15:52, 14 November 2012 (UTC)[reply]

Is it correct to say that- wind in case of forest fire bring flame from burning tree to neighboring tree and in this way fire increases rapidly, but this doesn't happen in the case of candle flame. Will same thing happen if we stick some candles in a row and blow wind ? Sunny Singh (DAV) (talk) 11:09, 14 November 2012 (UTC)[reply]

Yes, wind helps a forest fire to spread, but as Shantavira says, pre-existing heat is a major factor. It's worth considering a blacksmith's forge, where moving air from a bellows or similar is used to increase the intensity and heat of a fire.
  • Oil well fires are typically put out using the "Wind" from a large high-explosive blast: the shockwave from a high-explosive charge "blows out" the flame in exactly the same manner as you blow out the candle: Bigger flames require bigger winds to blow them out. --Jayron32 15:40, 14 November 2012 (UTC)[reply]


See also the fire whirl article. Count Iblis (talk) 17:27, 14 November 2012 (UTC)[reply]

  • The analogy is flawed because one single candle has nothing near the geometry of a forest. Pack a shoebox with upright birthday candles, light one, wait a few seconds for it to warm up, and then blow on it with a speed scaled to a gale force as if the candles were 60 foot tall trees. Get back to us after you extinguish the charred box full of burnt wicks and molten wax. μηδείς (talk) 05:36, 16 November 2012 (UTC)[reply]
... and even a single candle in a light draught burns much more rapidly, often spilling wax in the process. Dbfirs 07:21, 16 November 2012 (UTC)[reply]

Parrot Drone or Quadrotor

Is there any project that try to develop Parrot Drone or Quadrotor that can lift a person or two persons? What's the obstacles to built it? Is it motor problem or battery problem? Is Parrot drone can fly very stable? What if another two motor added horizontally, can it fly faster? Will these two motors cause disruption in flight stability? Thanks... roscoe_x (talk) 09:10, 14 November 2012 (UTC)[reply]

What's a parrot drone ? I don't think a quadrotor is particularly safe. If any one of the four rotors fail, I assume it then crashes. StuRat (talk) 09:36, 14 November 2012 (UTC)[reply]
Parrot AR.Drone apparently. No flying machine weighing more than a few ounces can get off the ground it runs from a battery.--Shantavira|feed me 09:52, 14 November 2012 (UTC)[reply]
Not so. The Sikorsky Firefly is a manned electric helicopter. For that matter, a bit more searching reveals the E-volo VC1 (see picture in the article), a manned electric quadcopter-style rotorcraft. That said, there are major design tradeoffs that make battery-powered helicopters completely impractical given current or near-term future tech, just as the quadrotor concept isn't really solving a problem in the field. — Lomn 15:45, 14 November 2012 (UTC)[reply]
I suspect that mostly it's a function of "no need" once you scale up to manned aircraft. Helicopters and gyrocopters are well-developed technologies that don't need the additional complexity that a four-rotor design introduces. For your final question, there's the compound helicopter, which adds horizontal thrust for additional speed -- the Sikorsky X2 is a recent example that reached 250 kts in level flight. There's also the V-22 Osprey, a tiltrotor. Both types of aircraft have considerably more engineering difficulties than traditional helicopters. — Lomn 14:43, 14 November 2012 (UTC)[reply]
I found an idea for a two-person quadcopter named The Skyflyer and a suicidal Chinese man. I think less is more when it comes to personal helicopters. Trio The Punch (talk) 19:36, 14 November 2012 (UTC)[reply]
It seems odd that those two quadrotor designs put the pilot above the rotors. This design is less stable and obscures the view of the ground, which is critical for landing. StuRat (talk) 19:55, 14 November 2012 (UTC)[reply]
Its probably much safer to sit on top of them in case of a crash landing and if the rotors are above you you'll need extra big and strong (=heavy) landinggear. Trio The Punch (talk) 22:20, 14 November 2012 (UTC)[reply]
Interesting answers, thank you. Especially the news about the farmer who built his own flying machine. roscoe_x (talk) 00:58, 15 November 2012 (UTC)[reply]
Some of those farmers are pretty innovative - see Richard Pearse. Zoonoses (talk) 07:10, 15 November 2012 (UTC)[reply]
This thing is pretty cool, but unmanned. Trio The Punch (talk) 15:08, 15 November 2012 (UTC)[reply]

chemical / molecular cross-section

What is the formula for the chemical reactivity cross-section of a molecule? The article cross-section only gives info about light absorption cross-sections and neutron cross sections. 128.143.1.142 (talk) 12:12, 14 November 2012 (UTC)[reply]

Reaction rates depend on the details of the chemicals involved as well as their physical states and abundance. In general, there isn't a single cross-section that would be useful for all molecules or all possible reactions, though you can use tables of things like standard electrode potential to determine what chemical reactions are likely to proceed. If you want more detail, you probably need to ask about specific molecules and reactions. Dragons flight (talk) 18:24, 14 November 2012 (UTC)[reply]

this is what is the compuound of Splenda, sugar and cloro form this, can we understand if it is a natural usefull compound for the human body?

1,6-dicloro-1,6-dideossi-β-D-frutto-furanosil 4-cloro-4-deossi-α-D-galattopiranoside o C12H19Cl3O8. — Preceding unsigned comment added by 195.110.143.127 (talk) 16:26, 14 November 2012 (UTC)[reply]

It looks like you've given the IUPAC name for sucralose. Splenda is the brand name of a sucralose-based sweetener that also contains dextrose and maltodextrin. Do the sucralose and Splenda articles answer your question? They both have referenced sections on health and safety. 209.131.76.183 (talk) 18:49, 14 November 2012 (UTC)[reply]

Artificial 2-toned lobsters

Is it currently possible to manipulate a lobster so that it looks like this one? Are we able to control the pigmentation selection process at the first cell division of the embryo (of certain species)? Is this technically possible in all multicellular beings (if you invest lots of time and money for research)? Are 2-toned humans theoretically possible? Trio The Punch (talk) 16:27, 14 November 2012 (UTC)[reply]

It's called heterochromia which in this case is caused by mosaicism, an early mutation in one cell after fertilization; but can also be caused by Chimerism, the merger of two embryos. There are people with such a condition, but due to differences in the splitting of cells in mammals and arthropods a symmetrical half-and-half mixture would be unlikely. See these images. Yes, a mad scientist could make one, but almost certainly not as striking as the lobster. μηδείς (talk) 17:04, 14 November 2012 (UTC)[reply]
As a rule, the kind of straight-down-the-middle color boundary so beautifully illustrated here is more common in arthropods. See gynandromorph for other such examples. (A Google Image search will be rewarding also) The split doesn't have to honor this midline boundary, however, even in arthropods. In mammals, the contribution of different cells in chimeras is pretty close to completely unpredictable. Wnt (talk) 18:29, 14 November 2012 (UTC)[reply]
Thanks a lot guys, very interesting links! Trio The Punch (talk) 19:14, 14 November 2012 (UTC)[reply]
Cthulu, before he escaped the lab
Note again that the lobster in this case is a mosaic, not a chimera. Humans can be both mosaics due to a mutation in an early embryonic cell or chimeras of fused non-identical twins. Interspecies chimeras have been created, such as this horrific house mouse/deer mouse chimera. Note the monstrous lack of symmetry in the eyes due to the developmental gene expression. Do that with a Human and a Chimp and you'll get something out of H. P. Lovecraft. μηδείς (talk) 20:33, 14 November 2012 (UTC)[reply]
I hope our mad scientist creates a Cthulhu-like creature. Trio The Punch (talk) 22:12, 14 November 2012 (UTC)[reply]
Yah, I've been daydreaming the human-chimp thing since the eighties... we could use an interpreter or two for interspecies communications. :) Wnt (talk) 22:44, 14 November 2012 (UTC)[reply]
Nyeah, bad idea. Reminds me of Asimov's The Ugly Little Boy. -- OBSIDIANSOUL 10:13, 15 November 2012 (UTC)[reply]

I don't know why would someone ever want to have 2 skin colored. It looked pretty abnormal and doesn't look good at all to me. Plus it is not ethical to do that anyway. You can't just test that on people.174.20.41.202 (talk) 21:58, 15 November 2012 (UTC)[reply]

Agreed, see wrongful birth and wrongful life. The picture of the chimerical mouse I linked to should scare the pants off any ethical experimenter. A human-chimp hybrid would be symmetrical and even-colored--but it's still not worth the risk. μηδείς (talk) 22:23, 15 November 2012 (UTC)[reply]

Named particles?

The Oh-My-God particle was a specific particle which was named for its surprisingly high speed. This is the first example I'd ever heard of a specific particle having a name. (Not a class of particles, but this particular proton.) Are there other examples? Staecker (talk) 16:35, 14 November 2012 (UTC)[reply]

Not a particle, but of the same idea: See Wow! signal. --Jayron32 17:09, 14 November 2012 (UTC)[reply]
This is strictly a case of notability. Anyone can arbitrarily name anything they want. For example, I name all of my protons, and some of my better mesons, but I'm not famous enough for any of my pets to have their own encyclopedia articles. Nimur (talk) 17:11, 14 November 2012 (UTC)[reply]
Yes I know. I'm asking if any other individual particles have achieved this level of notability. Staecker (talk) 00:15, 15 November 2012 (UTC)[reply]

gravity = side effect of vacuum?

Could it be that the displacement of vacuum by massive objects causes the density of the vacuum closest to the object to be denser than the vacuum any distace farther away from the object, thus causing the denser vacuum to "suck" smaller objects to that area, which just happens to be the surface of the object?165.212.189.187 (talk) 20:45, 14 November 2012 (UTC)[reply]

Gravity is the effect of mass causing curved space-time. (relax.... its non intuitive and complicated) Spacetime — Preceding unsigned comment added by Ap-uk (talkcontribs) 21:25, 14 November 2012 (UTC)[reply]
Thanks, (and no offense) but that sound like regurgitated double-talk165.212.189.187 (talk) 21:31, 14 November 2012 (UTC)[reply]
It's much easier to regurgitate sound-bytes describing our most non-intuitive physical explanations than to understand them! In any case, whether we understand it or not, gravity is a phenomenon that we observe in the universe. It is one of the most fundamental interactions we know, and it appears to contribute to the dynamics of almost everything we know about (with only a very few exceptions). We can describe the effects of gravity in many different ways. For many purposes, it is useful to describe gravity as a force, whose strength depends on a quantity we call mass. This is the "classical" gravity that physicists attribute to Isaac Newton. Einstein and more recent physicists used a different mathematical formulation to avoid the trouble with inexplicable action-at-a-distance, and to resolve some of the more subtle effects of gravity, like the way it interacts with electromagnetic waves. But, whichever formulation of gravity you use to model your observations, very few physicists ever talk about the "density" of a vacuum. And I have never heard any reputable physicist suggest that vacuums "cause" gravity as a side-effect; I can't really even figure out what that would mean. "Vacuum" is just a term we use to describe a region of empty space. It sounds as if you're trying to stretch an analogy of gravity to some type of buoyant force, and you're hypothesizing some type of gravitational phlogiston fluid - but if you carefully study all the implications of that hypothesis, I think you'll find that theory does not correspond to what we observe in nature. You are not alone; many great scientists throughout history have tried to use a hydraulic analogy to describe as-yet unknown effects of heat, electricity, and chemistry; but in each case, these hypotheses have been superseded by more parsimonious explanations. Nimur (talk) 22:26, 14 November 2012 (UTC)[reply]
Dammit, Jim, I'm a lawyer not a physicist, but 165.212.189.187 it seems to me that you've got the cause and effect wrong. Vacuum density is a measure of how much matter — gas and other particles — are within a space. Those things are denser near an object because of gravity, or to say it backwards, if it were not for gravity the density would not be higher near the object. Someone who's a real physicist will probably tell me I'm wrong, but that's my first impression. Regards, TransporterMan (TALK) 21:30, 14 November 2012 (UTC)[reply]


No offence taken , it's to do with perspectives. Mass, space and time are interlinked but our general default brain setting is one of 17th century mechanics. The proof of Einsteins curvature of space time explained 1) Mercury's weird orbit 2) Displacement of starlight around the sun. Both are shown here : Tests_of_general_relativity ... To show that time is not absolute a 1971 experiment is good fun :check out the "Hafele–Keating experiment" Ap-uk (talk) 23:00, 14 November 2012 (UTC)[reply]
  • One theory of gravity is that the vacuum exerts a positive pressure, and that the proximity of two bodies blocking that pressure from each other causes them to move together. I am not sure what that theory is called, so I can't get you references, but perhaps someone else here does. μηδείς (talk) 23:59, 14 November 2012 (UTC)[reply]
  • Three things. First, gravity exists and can be measured in the absence of vacuum or pressure forces, or even in the presence of opposing forces - a heavy weight dropped in the ocean will sink, even though the water pressure surrounding it in every direction is roughly equal, and that the water pressure increases with depth rather than decreasing. If low pressure (or vacuum) caused gravity's attraction, the weight would more likely float up into the sky. Second, attraction due to vacuum cannot explain how a small, very dense object like a black hole or neutron star can exert a much stronger gravitational effect on larger but less dense objects. This observation also affects the positive pressure theory Medeis mentioned. Third, because pressure in a given area tends to like staying equal, vacuum 'sucks' on an object in space roughly equally in all directions. With equal force pulling the object in every direction, the net force effect is zero, not 'sucked toward the nearest other object' as the question suggested. So in short, no, vacuum cannot explain the observable properties of gravity. NULL talk
    edits
    04:32, 15 November 2012 (UTC)[reply]
Arent atoms' volume 99% empty space?165.212.189.187 (talk) 16:35, 15 November 2012 (UTC)[reply]
@ Someguy and Null. Le Sage's theory of little particles is not the same one I was thinking of, but rather a Casimir effect caused by vacuum energy. The geometry is the same, but in the second theory it is a force, not particles. This is mention in this section of the Le Sage article, and no criticism or refutation is given. Second, the vacuum energy is someowhat of a misnomer. It's supposed to exist every, including in relative vacuums, not just in or because of vacuums. Perhaps the OP means vacuum in a different way from vacuum energy, he'll have to speak for himself. μηδείς (talk) 17:54, 15 November 2012 (UTC)[reply]
  • Just to be sure ... how does Le Sage's notion of ultra-mundane particles differ from the notion of virtual particles filling all space, e.g. a Dirac sea? If gravity is an exchange of virtual gravitons, is this the same as the blocking of exchange of certain virtual antigravitons which would be Le Sage's particles? Wnt (talk) 17:11, 16 November 2012 (UTC)[reply]

A question on economics.

Here in the UK a lot of economics statistics and also general work statistics (ONS) are used by the media who report it as fact.

Many years ago I got a degree in science and struggle to see how accurate these statistics actually are especially with economics. The general trend is to make it all look too difficult to the average person so they don't question things and also there is a lack of a glossary and explanation as to what has been left out or included.

1) Is economics a pseudoscience?

2) Is there some kind of bias in the statistics to make things look slightly less intimidating to the public? I notice for example that with regard to wealth stats the 69% of people who die in debt are left off the table, the wealth stats are then tabulated to take the super rich out (in fact you are in the top percentile with a net worth of only £423,000).

If economics is a pseudoscience why are no top scientists complaining or is it safer (as regards future financial science funding) to just pick easy targets like religious people ;)

--Ap-uk (talk) 21:19, 14 November 2012 (UTC)[reply]

1) Economics is a social science. 2) It would really depend on what you are looking at, who compiled the wealth stats you are looking at? They might have reasons to present data in a particular way other then "making it less intimidating".. And of course, nothing stops an economists biasing data if they have an agenda to push. Vespine (talk) 21:38, 14 November 2012 (UTC)[reply]
Your answer 1) is open for discussion. I wouldn't call it a science at all, social or not. Unless you are testing and improving your theories, do not call it a science. Science operate on the principle of falsifiability. The most amazing thing about economists is that they don't even care if their theory is wrong or not. It's an ideology pretending to be a science.~On the other hand, they indeed do use, although poorly, scientific tools like statistics, but that's not enough. OsmanRF34 (talk) 23:42, 14 November 2012 (UTC)[reply]
This is one of the sillier generalizations I have seen on here in awhile. There are indeed many branches of economics that involves compiling huge datasets and testing theories against them. There are branches of economics that intersect heavily with behavioral psychology and involve running clinical studies. It is a very broad field in terms of methodology. I have never met an economist who didn't care of their theories weren't correct. Saying "economics is an ideology pretending to be a science" is just reflective of your ignorance or prejudice and nothing more. --Mr.98 (talk) 15:44, 17 November 2012 (UTC)[reply]

I got the wealth stats from the ONS (Office of National Statistics) however it was not straight forward to find out the answers that I wanted and I had to glean it from several documents even then it was not 100% clear. The ONS did become separate from the government recently however I still see traces of "non-controversial" bias. — Preceding unsigned comment added by Ap-uk (talkcontribs) 21:55, 14 November 2012 (UTC)[reply]

This reminds me of the control systems lecturer we had when I was an electrical engineering student. He loved to poke fun at fields that were not like engineering, which is 100% based on fundamentals. He used to say that when an Engineering lecturer writes this year's version of the exam, he changes the questions. When an economics lecturer rewrites the exam for the year, he keeps the questions and changes the answers!
More seriously, economics when compared to things like engineering is a bit like psychology. Both economics and psychology DO have fundamentals, but the subjects are not fully understood and day-to-day issues have to be solved by postulation. Examples of fundamentals are: In psychology - all learning is either clasical conditioning (http://en.wikipedia.org/wiki/Classical_conditioning) or operant conditioning (http://en.wikipedia.org/wiki/Operant_conditioning); in economics, in a closed economy, the total value of money is equal to the total value of goods, services, and property. But can a psychologist predict what sort of responses will be triggered by my post, or can an economist predict how the price of widgets will react to Hurricane Sandy? Well, yes they can, but not by a process solely involving calculation and logical deduction starting with fundamentals. Engineering is quite different. In engineering, all solutions are derived from calculation and logical deduction starting with fundamentals like ohm's law. Then you get climate change science, which is not even as solid as psychology or economics. Wickwack 120.145.143.165 (talk) 01:13, 15 November 2012 (UTC)[reply]

The category of pseudoscience may not be as useful as you have been led to think. Classifying something as pseudoscience begs the question whether there is a generally applicable criterion by which to do so, the famous demarcation problem. Certainly no-one would argue that the economy is something beyond scientific study, and certainly most economists are quite earnestly trying to understand, quantify, and describe it. On the other hand, there are many scientists, including economists, who have made scathing critiques of the state of mainstream economics, for example that in economics, the theoretical side and the empirical side exist in fatal separation from one another (in a very interesting paper by an economist, but naturally I forget her name – I think she was from the University of Chicago). This, if true, would make economics bad science but not pseudoscience.

As to official government figures relating to the economy, you do well to take them with a good pinch of salt, since massaging the figures is usually the easiest way to make a problem go away. Inflation can be conveniently suppressed by adjusting the consumer price index, unemployment be kept down by defining people out of the ranks of the unemployed, etc. (I have no idea if and how much this is done in the UK, but would be surprised if it isn’t.) The “official” net worth of an individual person may not reflect their actual wealth with any degree of accuracy, since much of what the very rich own will often be nominally owned by some corporation or otherwise be distanced from them. But if you know how to read the figures correctly, they do provide useful information about real life. If the media report them uncritically, it’s the media’s fault, not the statisticians’.--Rallette (talk) 07:23, 15 November 2012 (UTC)[reply]

There are websites who specialise in checking out statistics, especially those quoted by politicians to back up their claims. check out Full Fact and Fact Check. (UK urls given as OP is UK) --TammyMoet (talk) 09:54, 15 November 2012 (UTC)[reply]
If there isn't enough information for you on how the official stats are collected, crunched and presented, there are always contact details of ONS staff who can answer your questions Some reasons why the methods might not be what you would regard as optimal include the need to ensure comparability with other countries, comparability over time, cost of collecting data and data protection. Journalists will never take any notice of the caveats that come with the data. Academic researchers should, but they also slip into professional conventions. The national productivity statistics are one of the most striking examples of the phenomenon you identify - they measure something, and they show that one country consistently "does worse" than another, but what they measure doesn't appear to be productivity. Itsmejudith (talk) 02:40, 17 November 2012 (UTC)[reply]


November 15

the material rising in the thermoscope \ thermometer

I would like to know if when the material rises in the the thermoscope meter (or in the thermometer), it indicates that the air in the thermoscope is shrunk or spreading. According to what I understand, the material (in example water ) spreading becaouse the warm and that is the couse of the material rising of the material inthe meter. What doyou think about? 46.210.166.245 (talk) 01:40, 15 November 2012 (UTC)[reply]

If the thermoscope is sealed, the liquid expanding will simply compress the air. If the temperature increases the air would also be warmed up at the same time as the liquid and if it was free to do so it too would expand, but since the liquid is far more dense then the air, the resistance the air applies to the fluid would be negiligable and the pressure inside the container would increase. I don't know for sure but I wouldn't be surprised if thermometers were created in such a way as to minimize the amount of air in the space before they are sealed. Vespine (talk) 02:36, 15 November 2012 (UTC)[reply]
I think it's more about the compressibility of a liquid (not very compressible) vs gas (easily compressible) rather than density itself as the underlying reason that the liquid expands to compress the gas even when ∆T says they should both be trying to expand. Our Alcohol thermometer article makes mention of the headspace above the liquid level. DMacks (talk) 07:10, 15 November 2012 (UTC)[reply]
Suppose, we have a thermometer of range 0 to 100 degree celsius. The thermometer is taken to a region of 100 degree celsius, I don't think that it will show 100 degree celsius if the glass cap of thermometer is attached at the same height as of 100 degree celsius mark. Air in the thermometer will compress to a certain amount, but will not get dissipate. If the cap is attached above the highest temperature scale (about 1 inch), the material inside is able to manage air compression and the thermometer will show correct temperature.
According to Vespine, pressure exerted by air on mercury will be negligible but what will happen if the mercury has expanded to the highest temperature. Will the air exert the same pressure at that temperature ? Air will also expand as the mercury (due to the heat of surrounding medium) and hence the kinetic energy of air molecules also increase and this time air will exert more pressure on mercury. Think about mercury barometer, it has vacuum in its mercury column but in case of mercury thermometer it has air. I do not know why ?Sunny Singh (DAV) (talk) 10:39, 15 November 2012 (UTC)[reply]
The air is confined to the volume left by the rising mercury. So the air pressure will increase due to 2 reasons 1) the pressure increases due to the reduced volume available, and 2) it increases due to the increased temperature. However, the increase in air pressure has negligible effect on calibration, as the mercury, being a liquid, is vitually incompressible. Floda 121.221.27.52 (talk) 11:14, 15 November 2012 (UTC)[reply]
I'm sorry, but I don't understand what does the meter shows when the material rises in the meter. the question is: Is the air in the meter is shrunk or spreading? (the thermoscope is open to get influences of atmosphere) 176.13.83.244 (talk) 18:17, 15 November 2012 (UTC)[reply]
The volume of the space at the top of the thermometer is smaller. That is the definition of "shrunk". --Jayron32 18:31, 15 November 2012 (UTC)[reply]
(edit conflict) As explained above, it doesn't make much difference whether the meter is sealed or open because it is the expansion of the material (usually coloured alcohol or mercury) that is being observed as the temperature rises. The only occasion when the compression of air is important is when a thermometer is heated well above its maximum range, and the air gets compressed almost to zero volume. The high pressure usually causes the bulb to explode. There may well be a partial vacuum above the material in a sealed thermometer, but this is not essential in a thermometer (though it is important in a barometer). Dbfirs 18:34, 15 November 2012 (UTC)[reply]
If you have a sealed container half full of fluid, when you heat it, the fluid will expand. The container with its contents will still have the same weight. But the fluid will be less dense, so another container within that fluid might sink. You should be able to make a Galileo thermometer which consists of concentric spheres, each containing a slightly different amount of fluid, so that each sphere rises or sinks within its surrounding sphere independently of the effect on all the other spheres. Though it would be terribly slow to respond unless made out of a very good heat conductor. I don't know if anyone has made such a thing but I would expect so. Wnt (talk) 02:52, 16 November 2012 (UTC)[reply]
Ever been to a gift shop Wnt? ;)
They are a bit on the expensive side but quite easy to find. Each glass orb has a punched sheet metal label, both to show the temperature and to calibrate the orbs. Clipping/filing the label will remove some weight and make the orb float more easily. - ¡Ouch! (hurt me / more pain) 09:27, 16 November 2012 (UTC)[reply]
I've seen them with multiple orbs in a single tube, which the OP wonders whether it is open or closed (which won't matter to the degree that the liquid is incompressible relative to the gas in the outer chamber anyway). But I've never seen them with one orb inside the next inside the next etc.; the tags would hinder the aesthetics of this and their weight would need to be well calibrated without them. Wnt (talk) 17:14, 16 November 2012 (UTC)[reply]
I thought that the OP was asking about a thermoscope (or ordinary thermometer), not a Galilean thermometer. Dbfirs 00:37, 17 November 2012 (UTC)[reply]
You're right - we had a different question about that one a little while ago... Wnt (talk) 05:43, 17 November 2012 (UTC)[reply]
Omg - I didn't get that you were talking about all orbs being concentric, not all merely within one outer containment. - ¡Ouch! (hurt me / more pain) 18:11, 17 November 2012 (UTC)[reply]

At the beginning of the statement, it says: "electromagnetic radiation (EM radiation or EMR) is a form of energy emitted and absorbed by charged particles". So are charged particles mean it is either positive or negative? How about neutral particle with no charge? Can neutral particle emit and absorb electromagnetic radiation?174.20.41.202 (talk) 03:17, 15 November 2012 (UTC)[reply]

Consider blackbody radiation. It can be emitted by matter that has, in bulk, no net charge. But, if you zoom in all the way to the fundamental interactions that are responsible for the thermal emission of radiant energy, you will discover that in the very microscopic sense, the radiation is emitted by a process that could be classically described as a movement of charged particles. We might say that warm objects have electrons in excited atomic orbitals, and that the photons are produced when the electrons decay. That explanation treats the photon as an emitted particle, and doesn't concern itself with the electrodynamics of the photon-as-a-wave. The exact details are actually somewhat subtle; for most purposes, we gloss over this detail; and if we want to study it closely, we have to use some very heavy-duty mathematical physics. Often, when we consider blackbody radiation, we are not interested in electrodynamic interactions; we only care about the energy that is being conveyed. But, any emitted electromagnetic radiation - whatever its source - is attributable to microscopic movement of charge.
When we discuss subatomic particles, we have to generalize the way we think about emitted radiation. Energy can be emitted in other forms besides electromagnetic waves, especially if it's radiated in a way that isn't related to wiggling electric charges. Sometimes, certain "exotic" energetic interactions between particles result in an emitted particle - which carries energy - that is not an electromagnetic wave. A neutrino can be emitted and can convey energy, without any interaction with electric charge. A phonon is energy that is emitted that takes the form of propagating wave in bulk condensed matter. There are many such examples where the radiated energy is not electromagnetic in nature, because the source of energy did not participate in any electromagnetic interaction. Nimur (talk) 06:19, 15 November 2012 (UTC)[reply]
I think Nimur is wrong. Consider a quantity of non-reactive gas at some finite temperature. The gas molecules, which are not charged particles are, according to the kinetic theory, continually flying about in the x, y, z tanslational axes and also rotating about these axes. The gas will be emitting, in all directions, heat in the form of infrared radiation, which is electromagnetic radiation. If there is incoming infrared radiation of total wattage greater than that being emitted, the temperature will increase - that is the gas will absorb energy by increasing the translational velocity of the molecules. If the molecules are not symmetrical in all x,y,z dimensions some of the heat energy will be converted into increased rotational movement. Some will be absorbed by lengthening the atomic bonds, thus increasing the "flywheel effect" Charged particles have no part in this beyond forming the atomic bonds, which are not changed by the increse in temperature. It is possible to have a "gas" in this thermodynamic sense consiting not of complete atoms or molecules, but instead consisting entirely of one sort of particle (electron, nuetron, etc), which may or may not be a charged particle. Floda 121.221.27.52 (talk) 11:08, 15 November 2012 (UTC)[reply]
Um, "Charged particles have no part..." Is completely wrong. If, as you say, there is "lengthening of the atomic bonds" then, by definition, that is a change in shape of molecular orbitals which involves a change in energy of electrons, which requires the electrons to change energy levels in some way, which requires the emission or absorption of photons, QED. It is inescapable: if photons are produced, something about the motion/location/energy of electrons changed (which "property" of electrons changed depends on your perspective, but the initial statement that all EM radiation originates in changes to charged particles is fundamentally true.) --Jayron32 13:50, 15 November 2012 (UTC)[reply]
Jayron, you did not mention clouds of sub-atomic particles. So, if you (& Nimur) are correct in saying charged particles are necessarily involved in the emission and absorption of EM radiation, then a cloud of nuetrons either cannot exist, or cannot have a temperature? I always thought that the Kinetic Theory of Gasses applies to sub-atomic particles as well as atoms and molecules when they are not forming a solid or liquid. (A cloud of particles should obey the KTofG if the following are true: Sum of particle volume << cloud volume; number of particles is very large; except in collisions, interactions are negligible; Number of collisions between particles is large compared with collisions of particles with container). Floda 60.230.192.18 (talk) 15:34, 15 November 2012 (UTC)[reply]
I don't follow the connection that is being made here between kinetic theory and thermal radiation. A "gas" of sub-atomic particles will always have a temperature and other thermodynamic properties, but it will only emit or absorb thermal radiation if the particles carry an electric charge or have components that carry an electric charge. Atoms and molecules have no overall charge but they still emit and absorb thermal radiation because they have charged components and hence they have an electric dipole moment. A "gas" of neutral fundamental particles would have a thermodynamic temperature but would not emit or absorb thermal radiation. Gandalf61 (talk) 15:57, 15 November 2012 (UTC)[reply]
Such neutral fundamental particles will have to be completely decoupled from electromagnetism, otherwise there will be higher order effects leading to the emission of photons. This means that the particles must be hidden sector particles that are completely decoupled from the Standard Model. And even then there will be extremely small effects mediated by gravity. Count Iblis (talk) 17:37, 15 November 2012 (UTC)[reply]
To expand on Iblis's point: neutrinos don't have any electric charge, and are not mediated by photons, so interactions involving neutrinos only involve exchange of particles like the W and Z bosons. If something neutrino does causes the release of a photon, it is only of the "higher order effects" that Iblis mentions: the weak bosons that neutrinos release interacting in some way with a charged particle, and THAT charged particle releasing a photon. It should be noted (not mentioned yet as a point of confusion, but it could be given the way the discussion is going) that even neutrons are not fundamentally neutral; they are composed of charged quarks, and thus are subject to EM interactions just as other charged particles are. --Jayron32 20:18, 15 November 2012 (UTC)[reply]

God, I'm so lost... Didn't expect the answers to be this lengthy. What you guys saying are beyond what I can understand, I'm just a curious ordinary person who doesn't know physic very well. Can you someone simply answer my question in simply way? Can neutral particle emit and absorb electromagnetic radiation? If it can then how the article only say "emit and absorb by charged particles"?174.20.41.202 (talk) 21:43, 15 November 2012 (UTC)[reply]

The answer depends entirely on what you mean by "neutral particle". If you mean "any particle with no net charge" then the answer is "It can still emit and absorb EM radiation if the particle is made up of smaller particles that themselves have electric charges". IF you mean "a fundamental particle which is not composed of smaller particles, and which is fundamentally neutral at all levels of organization" then the answer is "No, it cannot emit and absorb EM radiation" Thus, both the hydrogen atom and the neutron, which are each neutral but which are also each composed of smaller bits that themselves are charged (a proton and electron in the former, up and down quarks in the latter) CAN interact via electromagnetic radiation. However, a particle which is both indivisible and neutral, like a neutrino cannot. Does that make any more sense? --Jayron32 22:15, 15 November 2012 (UTC)[reply]
[Edit conflict]Electrically neutral particles that are composed of equal amounts of positive and negative charge emit and absorb radiation, so yes, neutral particles do, but only because of the equal presence of both kinds of charge which can emit or absorb the radiation. Does that help? -Modocc (talk) 22:23, 15 November 2012 (UTC)[reply]
Jayron and Modocc's explanation make perfect sense to me. So basically if an elementary particle that has no charge then they won't emit or absorb radiation.174.20.41.202 (talk) 03:22, 16 November 2012 (UTC)[reply]
This isn't exactly true, only a good approximation. Neutral particles can have magnetic dipole moments, even if they are elementary. E.g. the neutrino has a magnetic dipole moment, despite it being a neutral elementary particle, see e.g. here, and they can then emit electromagnetic radiation. This happens due to quantum effects. In quantum mechanics, if A can interact with C via B, then A will also be able to interact with C if B isn't present due to virtual B popping out of the vacuum and disapearing again. This is how the neutrino gains a magnetic moment despite not consisting of charged particles. Count Iblis (talk) 03:40, 16 November 2012 (UTC)[reply]

A related question: why don't neutral objects create EM fields all the time as they move about? (Like say, a human running and sprinting and then slowing down to catch his breath and then running again?) Individually, the objects are made of charged particles, so shouldn't they generate their own fields, that then would mostly cancel out? Photons would be emitted though. 71.207.151.227 (talk) 23:42, 15 November 2012 (UTC)[reply]

You're blasting out electromagnetic radiation right now, without doing anything. See Night vision device and Thermography for some applications of this. --Jayron32 23:55, 15 November 2012 (UTC)[reply]
They wouldn't cancel out because the positive and negative charges are not located at the same place. See electric dipole moment for a simple example of an overall-neutral system of charges that has a very real electric field about itself. Someguy1221 (talk) 23:50, 15 November 2012 (UTC)[reply]
Jayron is completely right.
On top of that, the human body is both a conductor and a capacitor. It might be a coil, if you look at the blood vessels which contain in a coarse approximation a conductive NaOH solution. A few feet of copper wire and some simple electronic components are sufficient to recognize other bodies with some accuracy. Nothing with a snowball's chance to stand in court, but... I tuned one of these to a jogger I encountered regularly, during years when I had time to kill and easy access to the components needed.
When he passed me, the circuit made a nice "blip" sound. There were some false positives, but less than one in 100. I had to retune the circuit while I was losing weight, tho. My own signature was screwing with its detection ability. Boy was she scared...
But that's about it. You can't do anything really evil, like electrocute her, or plant a neat "Kill your boyfriend" thought. But hey, there are other methods to get that done nowadays. - ¡Ouch! (hurt me / more pain) 07:49, 16 November 2012 (UTC)[reply]
NaCl, dammit. Self-whack. <°|>>><< - ¡Ouch! (hurt me / more pain) 08:32, 16 November 2012 (UTC)[reply]

Recharging smartphone

Why does it take up to an hour to go from 99% charged to 100% on my HTC One S? 67.243.3.6 (talk) 15:48, 15 November 2012 (UTC)[reply]

See Lithium-ion battery#Battery charging procedure. The charging is not linear, but consists of three stages. Dbfirs 17:45, 15 November 2012 (UTC)[reply]

Doppler effect: two moving objects at an angle

What's the formula for the Doppler effect if two moving objects are moving at an angle subtending the velocities between the two? For example, an airplane overtakes a motor vehicle on the ground-- how would you calculate the frequency shift of the engine roar emitted or received at any specific moment? 128.143.100.179 (talk) 18:41, 15 November 2012 (UTC)[reply]

First, you would subract the velocity vectors (assuming you have an external frame of reference) to get the relative velocity between the two points. Then you use the Dot product to project the velocity vector onto the vector between the two points. This will tell you how quickly the points are moving towards or aways from each other, which can then be fed into the Doppler formula. 209.131.76.183 (talk) 18:55, 15 November 2012 (UTC)[reply]
That's fine for light in vacuum, but the nonrelativistic Doppler formula takes the velocities of emitter and receiver relative to the medium. Their relative velocity isn't enough. -- BenRG (talk) 05:23, 16 November 2012 (UTC)[reply]
You could also use a full-wave-equation modeling technique, such as the Finite-difference time-domain method, to estimate the wavefield at every point. Then, you could sample the simulator to approximate the signal that would be observed for any trajectory at any velocity. This method is a lot more difficult to implement in practice, but if performed correctly, can account for many non-ideal effects, like the interaction between the sounds of the two vehicles; or imperfections in the propagation material. (Our article on FDTD has a strong bias toward the application for solving electromagnetic wave equations, but you can equally-well apply the numerical technique to solve the acoustic wave equation, and many other multidimensional PDEs). Nimur (talk) 20:05, 15 November 2012 (UTC)[reply]
Okay, I would have to do this in an exam. (It's a problem that I anticipate, as my professor likes to give us "climax of all that we know" questiosns. Also, according to my lecture notes, converting the two velocities into a relative velocity doesn't really work as v becomes on the same scale as c (e.g. 0.1c) . 71.207.151.227 (talk) 23:38, 15 November 2012 (UTC)[reply]
Use the lorentz transform to calculate the relative velocities when you suspect that the effects will differ from the classical case. Then, use that intermediate result and apply your favorite variation on the relativistic Doppler effect. It sounds like this work is related to a class; so check your notes or textbook for your professor's formula; or you can use the formula provided in our article. Nimur (talk) 00:23, 16 November 2012 (UTC)[reply]
I meant c as in the speed of sound, not the speed of light. 71.207.151.227 (talk) 22:28, 19 November 2012 (UTC)[reply]
A general formula for the nonrelativistic Doppler shift is
where
and are the emitted and received frequencies,
is the speed of sound,
is the velocity of the source at the time it emits the sound,
is the velocity of the receiver at the time it receives the sound,
is a unit vector pointing from the location of the receiver at the time of reception towards the location of the source at the time of emission,
and all the velocities and positions are computed in the rest frame of the medium.
It should be easy to see how this reduces to the one-dimensional case given in the article.
For the relativistic Doppler shift of light in a vacuum, there's a simpler formula: , where and are now four-velocities and is a four-vector pointing from the spacetime location of reception to the spacetime location of emission. You're no longer limited to a particular reference frame (of course) and doesn't have to be a unit vector (in fact, it can't be, because it's lightlike).
Should I add these formulas to the articles?
One could also derive a relativistic formula for sound or light in a medium, but I'm not sure how useful it would be. -- BenRG (talk) 05:23, 16 November 2012 (UTC)[reply]

Origin of the Nebraska sand hills

Hi. Currently, I'm researching the Sand Hills (Nebraska) for a world geologic location and hazard research assignment, and I'm exploring the geologic origins and proneness to mass wasting of these ancient sand dunes. What I've found so far is that the sand hills region is composed of Quaternary-age "Eutric" regosols, bounded by much sediment and/or uplift of Eocene to Mesozoic (think Niobrara Sea) age, located at the edge of the Denver Basin, with loess deposits extending to its east and southeast (yes, I have citations for all this info), reaching all the way to the Mississippi River and beyond, while the loess sediment also reaches down to the mouth of the Mississippi River - perhaps this makes sense, since the Platte River runs through the southern sandhills. I'm asking anyone for more relevant information about this area that they can find, including the depth of the sand (soil and rock core profiles would really help), the presence of any halite or gypsum salt deposits (I'm noticing that Nebraska is downwind of Lake Bonneville of similar age), and the effects of past explosive volcanic eruptions including at Yellowstone caldera on the region, and any evidence of past vegetation or absence thereof - for instance, the area's dunes were active during the Medieval Warm Period. Might the dunes creep east of Omaha and Lincoln, NE in the not-too-distant future? Also, what is the source of the sand dunes scattered throughout the Lake Michigan shoreline - are they direct deposits from the Laurentide ice sheet, as suggested by the USGS map on the "loess" article, or might the Nebraskan sandhills contributed to their origin sometime during the Medieval Warm Period? Any information on the influence on the stability of the dunes from the underlying artesian aquifer - the Ogallala Aquifer, which has its thickest portion underneath the sandhills and which is used for irrigation in mile-wide circular drip irrigation systems, which has dropped several metres in the past decades. Also, what about tornado alley - can you get sand tornadoes over devegetated portions of the dunes? What type of dunes are they - Barchan, Transverse, Longitudinal, Concave, Parabolic or Star? Is there any evidence of erosion by water on the dunes, and what are their permeability, porosity and density like? What are the differences and similarities between Nebraska's sand hills and other sand hills regions across North America and worldwide?

I'm also willing to contact certain universities and research/environmental organizations who might have more relevant information or current research on the topic, either by email or by phone - can you recommend any groups I might contact?

Thanks. ~AH1 (discuss!) 21:57, 15 November 2012 (UTC)[reply]

I found my answer to the Michigan sand dunes at Lake Chicago - by the way, remember that any extra research done here could also go into improving the article(s) discussed. ~AH1 (discuss!) 22:10, 15 November 2012 (UTC)[reply]
Answering as to the tornado part, tornadoes in the Nebraska sand dunes will act just as tornadoes anywhere else. As rapidly rotating columns of air, tornadoes are not necessarily visible; the only reason they are visible are if a condensation funnel forms or if they pick up debris from the ground. Most often the debris picked up that makes a tornado visible is dust/dirt (thus how you can have red tornadoes in Oklahoma). Because of this I would expect at least some dust/sand to be in any tornado over the sand hills, but this doesn't make them any different from other tornadoes. Ks0stm (TCGE) 01:22, 16 November 2012 (UTC)[reply]
I found out from this 32-page piece of research that the Sand Hills are composed of mostly complex transverse dunes, and that the aquifer does indeed indirectly stabilize the dunes: it feeds shallow wetland systems, that in droughts can release evaporation that sustains neighbouring prairie grasses. ~AH1 (discuss!) 04:45, 16 November 2012 (UTC)[reply]
Your apparent thesis that aquifer depletion could release these dunes and cause the desertification of the area is most intriguing, but as you've probably gathered by now, the Refdesk is having a lot of trouble rising to your level on this one (myself included). My gut feeling is that if you talk to someone in the right department at one of the colleges closest to this feature (no matter how small), you'll get far more information that we can provide. Though actually one source I found [2] which I think is saying the aquifer is not presently in trouble, as long as precipitation doesn't greatly decrease, was from a Chinese group. Wnt (talk) 18:24, 17 November 2012 (UTC)[reply]

Bok choy is the same species as the common turnip, really? It resembles nothing like a turnip

I know that artificial selection can really bring out the variety in a species, but this amazes me. How is it possible? It tastes nothing like turnip leaves. 71.207.151.227 (talk) 23:32, 15 November 2012 (UTC)[reply]

And yet a Chihuahua, a Newfoundland, a Boston Terrier and a Gray wolf are also all the same species of animal. It doesn't require your belief to be true, you know. The Brassica genus contains hundreds of different vegetables in about half a dozen species, and Bok Choy and Turnips are indeed, to the best of our ability to classify such things, the same species. --Jayron32 23:51, 15 November 2012 (UTC)[reply]
Sub question: Whats the importance or relevance of knowing that bok choy and common turnip are of the same species? (Of course im not asking specifically abouit bok choy and turnip alone) 203.112.82.129 (talk) 00:19, 16 November 2012 (UTC)[reply]
What's the point of knowing anything not immediately useful in daily life, really? But seriously, it could help with people trying to breed new strains to know that they can cross-breed related plants. It helps to keep from wasting time trying to narrow it down from all the other plants in the world. That's the first answer off the top of my head. I'm sure there are plenty of other reasons, too. Mingmingla (talk) 01:50, 16 November 2012 (UTC)[reply]
Trivial knowledge about various beet species provides for good conversational filler between more important, but perhaps less stimulating, quotidien tasks, such as programming iPads. On the statistical average, it helps us focus when we can rest our brains for a little while by analyzing simpler problems like taxonomic minutiae and behaviors of different types of muons. In other words, it's in the national interest to support beet research and discussion, even when the immediate benefits are not apparent. You wouldn't want to remove such stimulating discourse from our society, and through the consequent demotivation, preclude our most creative and productive minds from cranking out iPads at peak efficiency, would you? Nimur (talk) 02:35, 16 November 2012 (UTC)[reply]
Even now, sinister forces are hard at work trying to create an invincible army of bok choynips. Clarityfiend (talk) 01:54, 16 November 2012 (UTC)[reply]
And you'd be right. I used to work at an institute for fruit and vegetable research, and crosses like this are commonplace. Mainly to restore traits lost in the one with traits from the other, like disease and pest resistance. Dominus Vobisdu (talk) 01:59, 16 November 2012 (UTC)[reply]
My mistress' eyes are nothing like a turnip. --Trovatore (talk) 02:03, 16 November 2012 (UTC) [reply]
The knowledge might also be relevant since if you are allergic to one variety there's a good chance you'll be allergic to all. μηδείς (talk) 02:22, 16 November 2012 (UTC)[reply]
According to [3], Brassica rapa is a 6n (triploid) species. "The recurring genome duplications and triplication events have created massive genetic redundancy that quickly opens the possibility of sub-functionalization and neo-functionalization for duplicated or triplicated homeologs (Force et al., 1999; Shruti and David, 2005). It is likely that the extreme morphological diversity seen within the various Brassica species is due, at least in part, to the genetic redundancy and functional diversification permitted by these genomic events." Wnt (talk) 02:47, 16 November 2012 (UTC)[reply]
What Wnt's interesting statement means in less technical terms is that at some point the normal paired set of chromosomes found in the parent of the species got tripled (as if a human somehow got six sets of 23 chromosomes, in stead of just two sets), meaning that some of what were then extra genes got freed up to mutate and develop other functions that otherwise wouldn't have been possible if all the genes were busy serving their original functions. μηδείς (talk) 05:09, 16 November 2012 (UTC)[reply]
And, of course, we have an article on this!--Rallette (talk) 06:54, 16 November 2012 (UTC)[reply]


November 16

At the beginning of the article, it says: "the gravitational force appears to have unlimited range"? What is that mean? Let say the Sun's gravitational force and if I'm like a million or a billion light years ago from the Sun, can the Sun's gravitational force affect me? If not then the statement in the article is a false statement.174.20.41.202 (talk) 03:30, 16 November 2012 (UTC)[reply]

Yes, gravitation has infinite range, as far as anyone can tell. The laws of gravitation, either using Newton's law of gravitation or the models of general relativity predict that there is no limit to how far away two objects can be while still influencing one another. It is always the case, however, that beyond a certain distance you will no longer be able to detect the influence of some given object, as the attraction gets weaker and weaker. Someguy1221 (talk) 03:35, 16 November 2012 (UTC)[reply]
Newton's law of universal gravitation tells us the gravitational force between 2 objects is inversely proportional, not to the distance between them, but the square of the distance between them. That is, double the distance and the force diminishes by a factor of 4. Triple the distance, and the force is now only one-ninth what it was, and so on. So, if 2 objects are a billion light years apart, there's still a gravitational force between them, but it's infinitesimally small. Way too small to be measurable. But not technically non-existent. -- Jack of Oz [Talk] 06:22, 16 November 2012 (UTC)[reply]
LOL at some distance, it is pretty much like nothing... The concept in unlimited range is pretty mind blowing. 174.20.41.202 (talk) 07:09, 16 November 2012 (UTC)[reply]
That's what I meant by "way too small to be measurable". The difference is that, while we could not measure it empirically at all, we could still work out on paper what the infinitesimally small force would be if we could measure it, assuming the paper was large enough to hold all the zeros in the calculation. -- Jack of Oz [Talk] 20:57, 17 November 2012 (UTC)[reply]
You might be a little careful, given that you don't know whom exactly you're talking to, with the word infinitesimal. I take it that you're using it in the informal sense of "very very small indeed", and not in the precise sense of "smaller than any positive real number". --Trovatore (talk) 21:01, 17 November 2012 (UTC)[reply]
There is a limit ultimately, because according to general relativity, fluctuations in gravity actually travel at the speed of light, instead of infinitely fast as Newtonian gravity incorrectly assumes. So gravitating bodies outside of the observable universe have precisely zero effect on us, because the metric expansion of space keeps the gravitational effects of those bodies from ever reaching us. But the distance scales under consideration implicitly in the graviton article when it uses the word "unlimited" are vastly smaller than the diameter of the observable universe. Red Act (talk) 07:29, 16 November 2012 (UTC)[reply]
You are applying gravitational force on a number of sun(s) in other galaxies as well as on these galaxies, but they are not being affected by your gravitational force. It doesn't matter whether they are being attracted or not, but you are attracting them. Sunny Singh (DAV) (talk) 11:26, 16 November 2012 (UTC)[reply]

There are 6 kinds of lepton, one of them is electron. I know electron flying around the nucleus, which made up of neutron and proton or more precisely quark, in an atom. So where can you find the other 5 kinds of leptons?174.20.41.202 (talk) 07:09, 16 November 2012 (UTC)[reply]

The charged leptons in the second and third generations, i.e. the muon and tauon, decay rapidly, so they don't occur in normal matter, and are only seen in extremely high-energy environments such as cosmic rays or particle accelerators. The electron neutrino would be found in the highest concentration around radioactive matter undergoing beta decay. Neutrinos of all three generations stream throughout the universe, but rarely interact with normal matter. Red Act (talk) 07:47, 16 November 2012 (UTC)[reply]
If you could go with a neutrino, you could travel the universe, too, once you lepton, of course. StuRat (talk) 18:21, 16 November 2012 (UTC) [reply]

W and Z bosons are part of the gauge boson. While gluon and photon are massless then how can W and Z bosons have mass? Where are the W and Z boson come from? What emit and absorb them? 174.20.41.202 (talk) 07:23, 16 November 2012 (UTC)[reply]

Read W_boson#Weak_nuclear_force for your second question. Sunny Singh (DAV) (talk) 10:50, 16 November 2012 (UTC)[reply]
The W and Z gain mass from the electroweak symmetry breaking. The W+ and W- are the boson that is involved in beta decay. But all fermions interact with it. A Z0 can be produced in any interection that can produce a photon (as long as there is enough energy to form it's mass). Also just to clarify the W and Z are gauge bosons as are photons and gluons. Dja1979 (talk) 17:42, 16 November 2012 (UTC)[reply]

How's this bird called?

What's this bird? Picture: http://i.imgur.com/sDG5w.jpg 109.173.37.164 (talk) 07:31, 16 November 2012 (UTC)[reply]

European Robin (Erathacus rubecula) Richard Avery (talk) 07:58, 16 November 2012 (UTC)[reply]
Thanks. 109.173.37.164 (talk) 12:33, 16 November 2012 (UTC)[reply]
In the UK, it's a symbol of Christmas [4]. Alansplodge (talk) 10:56, 17 November 2012 (UTC)[reply]
Interesting. The Northern Cardinal is the Christmas Bird here in the US. We could export you some to go with the grey squirrels. μηδείς (talk) 19:59, 17 November 2012 (UTC)[reply]
Only if they're the St. Louis variety. ←Baseball Bugs What's up, Doc? carrots06:10, 19 November 2012 (UTC)[reply]
You may have trouble finding breeding pairs. μηδείς (talk) 17:36, 20 November 2012 (UTC)[reply]

Curiosity image

I think this image of Curiosity (found on found on this BBC page) seems to be a pretty sophisticated simulation - because I can't see anything like a boom extending from the rover out of the image that could be holding a camera.

Right? Your Username 08:35, 16 November 2012 (UTC) — Preceding unsigned comment added by Hayttom (talkcontribs) [reply]

It is a montage of pictures, see [5]], got from [6] Dmcq (talk) 10:05, 16 November 2012 (UTC)[reply]
Thanks very much. (There's a really interesting debate there.)
Resolved
Your Username 09:02, 17 November 2012 (UTC)[reply]

Is natural folate metabolized to 5-MeTHF?

"Folic acid" and "folate" are used interchangeably but are not exactly the same chemical compound and have some different properties.

Folic acid is metabolized to 5-MeTHF (aka 5-MTHF, 5-methyltetrahydrofolate, Levomefolic acid) by the human body.

Is natural folate also metabolized to 5-MeTHF?

Thanks. — Preceding unsigned comment added by 134.153.91.186 (talk) 15:31, 16 November 2012 (UTC)[reply]

The distinction between the two is only in dry form or on paper or in solutions when you can set the pH to what you want. Let folic acid, or folate, loose in an aqueous solution, and it can freely exchange away a proton (H+) to any passing water molecule, and pick one back up later on, at a very fast time scale. The counterion determines which form you see on the shelf in a lab, but as an ionic compound the folate and counterion are free to wander in aqueous solution, and in a biological solution there are so many other ions floating around for each to hang out with that they will act independently, never reuniting. Wnt (talk) 17:22, 16 November 2012 (UTC)[reply]
Thanks, Wnt, but I may have given the impression that I know more chemistry / biochemistry than I actually do. Does your answer mean, "yes, natural folate is metabolized to 5-MeTHF"? If I eat broccoli and bread in the same meal, will the folate from the broccoli and folic acid from the (fortified) bread undergo the same processes in my gut and/or elsewhere? — Preceding unsigned comment added by 134.153.91.186 (talk) 22:00, 16 November 2012 (UTC)[reply]
Yes. Wnt (talk) 02:53, 17 November 2012 (UTC)[reply]

What happens to the brains of the enlightened?

Is there some serious study about the effect on the brains of the enlightened (in a spiritual way)? If they feel different something must be different somewhere, I suppose. OsmanRF34 (talk) 17:02, 16 November 2012 (UTC)[reply]

[7][8][9] ... (the secret to getting these results is only to know the keywords "religiosity" and "fMRI" for a Google search, which spit them out as the first three of a long list of similarly relevant results!) Wnt (talk) 17:25, 16 November 2012 (UTC)[reply]
I wanted something more specific. Instead of 'religious' something that's the product of meditation. And instead of anatomy of the brain, something at the level of neurotransmitters and hormones, which is what makes us feel. Unhappily, a simple search for 'neurotransmitter' and 'meditation' pours out a wealth of dubious pages with dubious claims. OsmanRF34 (talk) 18:10, 16 November 2012 (UTC)[reply]
Oh Lord, first you're going to have to define enlightenment. I mean, you have to know who to put in your fMRI machine before you can look for differences in the pattern. Trying "Monk" and "fMRI" I get [10] which talks about soon to be published research by an adjunct professor ... surprisingly, here it is. I'm going to leave it at that for now - my wheels are spinning freely when I try to skim through this one. Wnt (talk) 19:25, 16 November 2012 (UTC)[reply]
Wnt: your "wheels are spinning freely" because you lost track in this and the question below. I never said religious, and I clearly say neurotransmitters and hormones. OsmanRF34 (talk) 21:23, 16 November 2012 (UTC)[reply]
I don't know if you're interested in this report, as it's on brain scans in psychographers (automatic writing). But it may give you a place to do further research. --TammyMoet (talk) 19:34, 19 November 2012 (UTC)[reply]

Sleep more to grow more

I have read somewhere that our height increase only when we sleep. What do you think about this ? Is it true ? Sunny Singh (DAV) (talk) 17:10, 16 November 2012 (UTC)[reply]

Sitting, standing, etc. compresses the spine and reduces height. [11] So normally for most people height will decrease during the day and increase at night. This change in height will be much more than the change due to growth. What I don't know is whether more time in a prone position increases the rate of long-term growth - I can't think of a reliable way to do the experiment to look it up. Also I should add that the suggestion the paper I cited seems to be making about hyperextension as some sort of prophylaxis for occupational stress sounds nuts to me, and from a quick glance I don't think they prove it in any way; I suspect it should have some harmful effect on the discs if any at all. Wnt (talk) 17:34, 16 November 2012 (UTC)[reply]
I think the OP is asking about development from birth to adulthood, not decompression of the spine. The consensus is that sleep is when most growth occurs, and that disturbed sleep can cause stunting. Since sleep is a necessary but not causative factor getting enough sleep in for the days growth is important, but extra sleep won't lead to extra growth. See this article for sleep and puberty, and these for growth during sleep. If your own or a specific person's height concerns you, see a doctor. There are various causes and treatments for abnormal shortness, and abnormal tallness can be a sign of dangerous but treatable conditions. μηδείς (talk) 18:09, 16 November 2012 (UTC)[reply]
I think Wnt was saying it's difficult to study when growth occurs, since it's so overwhelmed by spinal compression. Also, even if you could establish that people who don't get much sleep are shorter, that doesn't necessarily mean the sleep is the causative factor. It might be that whatever stresses keep them from sleeping also stunt growth. StuRat (talk) 18:15, 16 November 2012 (UTC)[reply]
Actually, our article on sleep mentions a study of children finding no correlation between growth rate and the amount they slept. But as StuRat says, there are so many complicating factors... besides, just because the amount of sleep doesn't affect the amount of growth still doesn't prove the growth can't occur mostly when one is asleep. Wnt (talk) 19:13, 16 November 2012 (UTC)[reply]
Reference 10 of Wikipedia article Growth hormone says growth hormone is secreted during sleep. 'Biological regulation' section of the article growth hormone mentions deep sleep as stimulator of growth hormone. I think there is correlation between growth rate and sleep, but I am sure, so I have asked it here to get the correct answer. Thank you. Sunny Singh (DAV) (talk) 00:29, 18 November 2012 (UTC)[reply]
The question is not: if we sleep more, do we grow more. It's only whether we grow while we sleep, which seems to be answered by Medeis above. OsmanRF34 (talk) 21:27, 16 November 2012 (UTC)[reply]
Actually I don't think the question really is answered; it's complicated. Apparently in culture bone formation is enhanced by intermittent and even to some degree constant compressive force (such as one would expect while awake) - see PMID 3505768, PMID 22559784 - yet it also increases apoptosis (PMID 16368547). But the actual process of sleep, rather than inactivity, is a more complex phenomenon that can't be replicated in a dish, and I didn't find anything on the effect of it on extracellular matrix deposition or other effects in cartilage in a quick NCBI search. Wnt (talk) 18:04, 17 November 2012 (UTC)[reply]
I always see people on reference desk using OP while they answer, but I don't understand its meaning. Please, tell me its full form. Sunny Singh (DAV) (talk) 04:36, 17 November 2012 (UTC)[reply]
It is the Original Poster, the Other Person. It comes from internet talk, amounts to a polite genderless way to refer to someone already mentioned. If you see the IP it means the person editing with an IP adress as a user name. In the case above, since you and I haven't really interacted, and since Wnt should realize I was addressing him/her, and talking about you, using OP was not unusual. Had I said "I think Sunny..." it would have implied more certainty on my part in this case. But Sunny would have been fine. I like having more pronouns to use to avoid ambiguity and to make subtler distinctions. Our lack of a formal tu-vos distintion is annoying, but one can say yourself/yourselves and you all. Excluding you request for clarification and my answer, the term is shown used five times on the board as of this edit. In the future, google OP urbandictionary to find out the meaning of any new usage, cutural, internet, adult themed--it's all there. μηδείς (talk) 05:23, 17 November 2012 (UTC)[reply]

Acceleration and time dilation

Okay, about a week or two ago I asked about the maximum G-force a person could withstand and a very helpful user told me we can withstand an acceleration of 1g for months on end and could even theoretically speed up to the speed of light in this manor (neglecting the problems surrounding mass and energy obviously).

Now, my question is this. As you speed up, due to time dilation, time for you, relative to an observer travelling at a constant velocity lower than yours, slows down. Acceleration is a vector quantity and is calculated using time.

If you were in a space ship accelerating they could calculate your acceleration using basic suvat equations. But would they have to take time as the time they experience pass or the time you experience pass. And if it is based on their time can you withstand 1g acceleration by their time or is this measurement based on their time? — Preceding unsigned comment added by 109.153.175.182 (talk) 19:13, 16 November 2012 (UTC)[reply]

1 g refers to the acceleration in the spaceship's frame of reference. Ruslik_Zero 19:37, 16 November 2012 (UTC)[reply]
(EC) Humans are comfortable with a 1g acceleration in their own frame of reference, i.e. a 1g proper acceleration, which is observed as being an acceleration of less than 1g in any inertial frame of reference in which your direction of acceleration is in the same direction as your velocity. You don't use the SUVAT equations, because those are Newtonian. Instead, the relativistic equation of motion for an object with constant proper acceleration is , as per the article Hyperbolic motion (relativity). Red Act (talk) 19:41, 16 November 2012 (UTC)[reply]

Thank you! This is extremely useful, however I am unfamiliar with this equation. What does the alpha and the x represent?

α is the acceleration, x is the position of the rocket (measured in some inertial frame), and t is the coordinate time measured in the same frame. The elapsed time for the person on board the rocket (proper time) is , where sinh is the hyperbolic sine.
Whoever said that people can withstand 1g for "months on end" was presumably joking. Anyone who lives at sea level withstands 1g acceleration for their whole life. But if, hypothetically, a person could only stand a particular acceleration for a few months, that would be a few months of proper time (tau). You could plug that value of tau into the reverse equation to get t, then plug that into Red Act's equation to get x (which is the distance traveled, more or less—actually, it's the distance traveled plus c2/α). -- BenRG (talk) 01:20, 17 November 2012 (UTC)[reply]

Okay, so what is the absolute limit of acceleration a person could withstand? Also by x being the position of the rocket measured in an inertial frame, do you mean some measurement of distance? — Preceding unsigned comment added by 31.54.166.31 (talk) 23:34, 17 November 2012 (UTC)[reply]

Also which time is T and which is t? — Preceding unsigned comment added by 31.54.166.31 (talk) 23:38, 17 November 2012 (UTC)[reply]

I think you mean tau (τ), not T. Tau is the proper time and t is the coordinate time. -- BenRG (talk) 03:08, 19 November 2012 (UTC)[reply]
Unfortunately the body's tolerance for acceleration does not seem to be the limiting factor. At 1g, we could get to foreign star systems in quite reasonable (ship's) times, assuming you're willing to spend years in a tin can. But first, no one knows how to achieve sustained acceleration of 1g. And if we did know a way, we still don't know any way to shield the occupants from the interstellar medium, which at relativistic speeds appears to you as hard radiation.
Larry Niven addressed these problems in some of his works (both in Known Space and otherwise) by some fairly optimistic hypotheses about the workability of the Bussard ramjet). Unfortunately, according to our current state of knowledge, those do not appear to be true. --Trovatore (talk) 02:22, 18 November 2012 (UTC)[reply]

This is extremely useful information. When you say "no one knows how to achieve sustained acceleration of 1g", is this because it would require unfeasible amounts of power or are there other reasons? Also are there any references for this interstellar medium appearing as hard radiation? These Larry Niven links are extremely interesting by the way, thank you for the reference/ — Preceding unsigned comment added by 31.54.166.31 (talk) 17:46, 18 November 2012 (UTC)[reply]

Because of conservation of momentum you have to push something backwards to accelerate forwards. Your choices are fuel you bring along (a rocket engine), the interstellar medium (a jet engine), or some large astronomical body. This page explains the problem with the rocket, which is that it needs a ludicrous amount of fuel. The problems with a jet engine (Bussard ramjet) seem obvious, but I don't know the details. In the third category there's laser propulsion, which would only be feasible at the beginning of the trip and probably not even then, and gravitational slingshotting, which probably isn't relevant to this thread.
The article I linked explains the radiation problem also, at the very end. -- BenRG (talk) 03:08, 19 November 2012 (UTC)[reply]

Neurons in the cochlear nuclei

How many neurons are there in the cochlear nuclei? — Preceding unsigned comment added by 144.96.215.130 (talk) 21:52, 16 November 2012 (UTC)[reply]

The book Hearing in Children, by J. L. Northern and M. P. Down, gives a total number of 8800, but doesn't make it clear how that number was obtained. Looie496 (talk) 00:04, 18 November 2012 (UTC)[reply]


November 17

Two problems involving angular motion

I do not know how to approach these problems. The first is finding the initial angular velocity/speed of a wheel that rotates 5.3 radians to a complete stop in 3.4 seconds. The second is finding the angular acceleration of a 26 centimeter thick 2.9-kilogram cylinder that has a force of 16 Newtons being applied to it. --Melab±1 05:02, 17 November 2012 (UTC)[reply]

The first problem isn't well-defined as stated; what's missing is presumably an assumption that the wheel is subjected to a constant torque, or equivalently, that it undergoes a constant angular acceleration. This problem might be easier for you to solve with a well-chosen change in variables. Think of a movie of the decelerating wheel that's played at the same speed as it was recorded, but backwards. The movie being played backwards will still be 3.4 seconds long, and will still show the wheel rotating 5.3 radians, still at the same constant angular acceleration except for a change in sign. What constant angular accelereration will produce a rotation of 5.3 radians in 3.4 seconds? And then given that value for the constant acceleration, what will be the speed of the wheel at the end of the reversed movie? Red Act (talk) 05:59, 17 November 2012 (UTC)[reply]
The basic procedure for the second problem is to look up the cylinder's moment of inertia at List of moments of inertia, determine the torque from the equation at Torque#Moment arm formula, and then find the angular acceleration from the equation at Angular acceleration#Constant acceleration. Red Act (talk) 06:19, 17 November 2012 (UTC)[reply]
However, we are missing some geometry info on the cylinder. Is that a hollow cylinder ? If so, and it's rotating about it's axis, we need an inside radius or diameter and an outside radius or diameter. Given that we have the thickness, we could find either one, if we had the other. We also need to know if it's made of a uniform density material. The length of the cylinder is not important, since we are given the total mass (it would be, however, if we were instead given the density). And where is this force applied ? Tangent at the outer diameter ? StuRat (talk) 22:32, 17 November 2012 (UTC)[reply]

civil engineering related

why moment of inertia of a hollow circular cross-section is more than a solid circular cross-section? — Preceding unsigned comment added by 49.137.37.116 (talk) 13:14, 17 November 2012 (UTC)[reply]

The weighted average distance from centre of all small mass elements making up a thin ring is equal to its radius, but the wesighted average distance from centre of all small mass elements in a solid cylinder must obviously be less that its' radius, as elements exist from the centre outwards. However, this does not mean that the moment of inertia will be less for a solid cylinder as its' total mass will be considerably greater. See the formulae in the List of Moments of Inertia article. Floda 121.221.79.138 (talk) 13:53, 17 November 2012 (UTC)[reply]
The moment of inertia for a single point is mr^2. Turning that point into a ring, or a cylinder, doesn't matter - you still have the same mass, rotating around the same axis at the same length. But making that ring solid means you take the mass that was at distance r, and distribute it in a column from the center point (moment of inertia zero) to the full distance r. However - the distribution is not even, because there's only one point at the very center. So a small segment of the ring, when distributed toward the middle, forms a narrow triangle, with the average point in the triangle at a distance of r over the square root of 2 away from the center. Twisted any which way, the result is half the moment of inertia. Wnt (talk) 20:49, 17 November 2012 (UTC)[reply]

if the eye changes focal length to focus on an object, why isn't our visual perception like a zoom lens?

I don't notice my visual field at infinity (f=17 mm) being that much wider than my visual field at f=22mm, at the minimum focusing distance, at least not on the order that I would expect a 30% change in focal length to achieve. 71.207.151.227 (talk) 17:45, 17 November 2012 (UTC)[reply]

That's because your eyes are not cameras and your brain is not a .jpg file. Visual perception doesn't really work like a camera. Of course, the laws of optics still apply to the lenses in your eyes, but beyond that how you perceive the electrical signals that get sent from your retina to your visual cortex is very different from what any camera does, and you can't really reliably draw analogies between the two. --Jayron32 19:46, 17 November 2012 (UTC)[reply]
  • In the eye, the image plane (retina) never moves in relation to the aperature (iris). Only the shape of the lens changes. Since the light moves in a relatively straight line from part of an external feature to a spot on the retina, the external feature remains about the same size. In a camera with a zoom lens, the distance from a lens to the film changes, but the lenses themselves are of solid and unalterable glass. Wnt (talk) 20:33, 17 November 2012 (UTC)[reply]
    A zoom lens is not "solid and unalterable glass," but a collection of glass lenses which changes the spacing between elements to alter the focal length. The distance of some elements to the film may change both to alter the focal length and to adjust the focus. Edison (talk) 02:11, 18 November 2012 (UTC)[reply]
    But if the focal length changes, the angle of view should change shouldn't it? A focal length is a measure of how strongly it bends rays overall, which in turn affects angle of view. It shouldn't matter by what method the focal length is changed. 71.207.151.227 (talk) 09:51, 18 November 2012 (UTC)[reply]
        • I think NorwegianBlue has put his finger on it, below. To rephrase, there are two variables involved — focal length and magnification. A zoom lens changes both; your eye mostly changes just the former. --Trovatore (talk) 02:32, 19 November 2012 (UTC)[reply]
  • A more relevant comparison with a camera would be using manual focusing, which does not change the size of the image, but allows you to choose whether the foreground or the background should be in focus. --NorwegianBlue talk 22:36, 17 November 2012 (UTC)[reply]
You've only got decent vision in your fovea anyway, so width of field is sort of irrelevant. Gzuckier (talk) 02:29, 19 November 2012 (UTC)[reply]
But the question is, what solid angle of the world is represented in that image on the fovea? If you could increase the magnification of your eye, then you could reduce that solid angle, zooming in on smaller features in the world. But you can't, to any significant extent. --Trovatore (talk) 02:38, 19 November 2012 (UTC)[reply]

Doesn't a zoom lens move a heck of alot more in relation to the film than the focal point of the eye's lens in relation to the retina?


Another opportunity for me to make a pitch for the excellent book, Applied Photographic Optics. Camera optics provide a user-interface that isolates focus and zoom as distinct entities - in other words, it is possible to focus, without changing field of view; and zoom, without losing focus. Very nice, expensive camera lenses can be focused with negigible parasitic zoom because they are complex multi-element glass. In my twelve-element 70mm-300mm telephoto lens, the "zoom" is actually driving a gear system that slides several elements - to correct for some optical aberrations, and to help stay mostly focused while continuously extending the focal length over two and a half octaves. If you have a simple lens, say a single element convex magnifier-glass - and you move it relative to the image plane to provide focus - then you'll also see a very obvious zoom or magnification effect. Several multi-element lens designs are discussed in the book, portions of which are available to browse for free online. Chapter 30.2 discusses practical considerations for distortion-free telephoto zoom. Nimur (talk) 13:25, 19 November 2012 (UTC)[reply]

November 18

Where is the Heisenberg cut article?

I searched for Heisenberg cut but didn't find it. So does it exist as a subsection somewhere or is it just a dead cat? Hcobb (talk) 01:10, 18 November 2012 (UTC)[reply]

There is certainly enough information out there to create an article: see [12]. Wikipedia only exists because people no different than you created and expanded all of the articles. That is, if you find something is missing from Wikipedia, you are literally the best person in the world to add it; or you're no worse than anyone else. So, feel free to create that article! --Jayron3201:44, 18 November 2012 (UTC)[reply]
Excellent, by the way Hcobb, on creating the article. This looks like an excellent candidate for WP:DYK if you can expand it a bit! --Jayron32 12:38, 18 November 2012 (UTC)[reply]

At the moment the article is (intentionally) isolated with no inbound links, but I hope it can either serve as the main article for the subsections in other articles that it notes, or be merged into a slightly more broad article that covers the concept. If so it might well be split between Heisenberg's own writings and other contributions to the subject, and the more general concept that has been taken up by quantum computing. Hcobb (talk) 17:49, 18 November 2012 (UTC)[reply]

BTW, suppose that you did have a Heisenberg cut device, that I suppose would measure all possible quantum numbers inside a box. Say you subjected an average sized household cat to this, how big would the resulting explosion be? A chemical explosive because you've just messed up all the electron orbitals? A nuclear explosion because you've swatted all the nucleons? Or an anti-matter level explosion because you've sampled all the quark states? Hcobb (talk) 17:59, 18 November 2012 (UTC)[reply]
Based on this question and what you wrote in the new article, I think you don't understand what the cut is. The cut is the correspondence you define between quantities in a theory that you want to test and quantities that you know how to measure in the real world. To put it another way, it's the boundary at which you splice a speculative theory to a commonsense engineer's physics that everyone already accepts as true. This is a necessary part of any theory that claims to describe the real world. To a large extent it doesn't matter where you place the cut, but it needs to be at a high enough level that all the controversial physics is handled by your theory.
A nice non-quantum example is Einstein's 1905 paper on special relativity. The math of special relativity had already been worked out at that time. Einstein's only contribution was giving it an interpretation that made sense. He did that by moving the cut. Instead of assuming that distances and time intervals are things that can in principle be measured to arbitrary accuracy by sufficiently good instruments, as everyone had assumed in the past, he treated those instruments as part of the system to be modeled by the new theory. As his measurable quantities, instead of position and time, he used the time shown on a clock face when some event happens at the clock's location. The assumption that a human experimenter (his "observer", though the meaning of that term has changed since then) can read a clock face is much weaker than the assumption that an experimenter can measure true distances and times. And it turned out that the latter assumption was actually wrong.
In quantum mechanics, if you make the usual division between quantum system and measurement apparatus, the disappearance of the interference pattern in the double-slit experiment seems bizarre. If you instead do as Einstein did and make the apparatus part of the quantum system, using readings on dials as your measurable quantities, you get a much more comprehensible result: the wave function gives a classical probability for each possible readout, the collapse is just a Bayesian update, and the disappearance of the interference pattern is caused by a quantum interaction between the photon/electron and the apparatus (quantum decoherence) which couldn't be modeled when you assumed the apparatus wasn't quantum.
By the way, I've never heard the term "Heisenberg cut" before. I've seen occasional mentions of the cut between theory and experiment, but not with Heisenberg's name attached. It does get a fair number of Google hits, but I think it's an unfamiliar term to most physicists and is probably used mostly by people like Stapp (who I consider a crank). -- BenRG (talk) 21:05, 18 November 2012 (UTC)[reply]
I agree with BenRG. I've read more than a few books on atomic physics, and I've never encountered this phrase in the context of quantum physics. From a cursory glance at the references in our new article, it appears to be a sort of popular science neologism used by "physics-philosophers." Nimur (talk) 12:34, 19 November 2012 (UTC)[reply]
To be entirely fair, being quackery doesn't mean it isn't proper article fodder. It may be quackery, and if so, the article should use reliable sources to report it as such. But quackery alone is not enough of a reason to not have an article. We have articles on homeopathy and astrology; the articles both report the "internal logic" of the beliefs and also explain clearly and unambiguously that the scientific community thinks they are bullshit. I have absolutely no familiarity with the term either, but I also don't have the physics background that you two do to find sources that refute it. If the rather stubby article needs expansion, one needed route of expansion may be some explanation (properly cited) of this as a not-accepted thing. --Jayron32 13:54, 19 November 2012 (UTC)[reply]
The final reference in the article doesn't look like quackery [13] - it describes specific writings by Heisenberg in the 1930s about contextual hidden variable theory, which he considered by postulating various placements of a cut "Schnitt" between observer and observed. I'm not likely to answer this question, but at least it is a valid concept from the development of quantum mechanics. Wnt (talk) 19:12, 19 November 2012 (UTC)[reply]
Just to clarify. I'm not saying one way or the other if the concept is quackery or not. The concept could be sound science, or it could be bullshit. Being bullshit is not, of itself, however, grounds for not having an article on something. Homeopathy is unmitigated bullshit, but that doesn't mean Wikipedia doesn't have an article describing it. Which is not to say that this concept is bullshit. It may not be; just that Ben and Nimur's objections to it are not grounds for not having an article about the concept. --Jayron32 20:19, 19 November 2012 (UTC)[reply]
I am also somewhat concerned that the article currently states "there are no actual Heisenberg cuts anywhere...". That last reference describes the Heisenberg cut as an integral part of the Copenhagen interpretation, so that statement appears to assert that Copenhagen is wrong. Copenhagen is certainly not as popular as it used to be, but I do not think it has been so soundly rejected as to allow an NPOV article to assert that. --Trovatore (talk) 19:32, 19 November 2012 (UTC)[reply]
Whoops, sorry, not the same ref -- I'm talking about this one. --Trovatore (talk) 19:35, 19 November 2012 (UTC)[reply]
I agree - we cannot be certain that Heisenberg cuts do not exist. They are required in some interpretations of quantum mechanics, but not in others. I have tried to make the article more NPOV. Gandalf61 (talk) 09:46, 20 November 2012 (UTC)[reply]

Voltage across an inductor

I read about the self inductance phenomenon and about the equation V=L(di/dt). But I am confused about the direction of V. My confusion is that current flowing through an inductor causes a 'back-emf', and the back emf is equal to V(or so I think). If so, shouldn't the direction of V be in the opposite of that of I? Also, in the Voltage and Current graph sketch across the inductor, I saw that the phase difference is 90 degree, but the voltage and the current are infact, in the same direction. I just don't get the fact that how V is in the same direction of the current that causes it. According to Lenz's law, it should be in the opposite direction. — Preceding unsigned comment added by 210.4.65.52 (talk) 04:17, 18 November 2012 (UTC)[reply]

Voltage doesn't have a direction; I guess you're talking about the direction of the voltage drop, which is what's meant by V. I don't know if this helps, but in the hydraulic analogy, an ideal inductor is like a heavy but frictionless paddlewheel. Its inertia opposes any attempt to either increase or decrease the current, which leads to a downstream pressure drop (positive V) or gain (negative V) in the case of a forced current increase or decrease respectively. -- BenRG (talk) 07:23, 18 November 2012 (UTC)[reply]
Another point to consider is that the induced voltage depends on the _change_ in current, not the static (DC) current. If you have a perfect inductor carrying a DC current, it won't have any voltage across it (because its DC resistance is zero). If you increase the current, the induced voltage will be in the "opposite direction" to the DC current - if you reduce it, the induced voltage will be in the "same direction". Tevildo (talk) 21:14, 18 November 2012 (UTC)[reply]

URGENT INFO NEEDED

Dear all, I need to know how thymoquinone actually acts! I mean does it pass through the cell membrane? Does it have a receptor? How does it expert its effect? attaches to a protein to change its activity? ... Any info regarding that will be appreciated. Best kukubah 04:51, 18 November 2012 (UTC) — Preceding unsigned comment added by Kukubah (talkcontribs)

PubChem is a good place to start for stuff like this (see here). Also, note the warning at the top of this page about timeliness - this really isn't the best place to ask urgent questions. Zoonoses (talk) 05:46, 18 November 2012 (UTC)[reply]
Hmmm, oddly enough I'm finding a source that the antinociception and anticonvulsant effects work via the kappa opioid receptor. [14][15] Wnt (talk) 06:04, 18 November 2012 (UTC)[reply]

When did dark energy start to dominate?

There's a graph currently on the Hubble's law article that shows universal expansion. It's hard to read the inflection point, and I don't find it mentioned in the article or any related articles.

That is to say: after inflation ended, the universal expansion was decelerating. Now it is accelerating. When was the critical time when deceleration became acceleration?

(Let's say, treating the dark energy as a cosmological constant... do other proposals imply a different critical time?) — Preceding unsigned comment added by 174.118.1.24 (talk) 08:07, 18 November 2012 (UTC)[reply]

Ignoring radiation (which only matters at early times) and assuming the universe is flat (no global curvature), the evolution of the cosmological scale factor is:
Where is the scale factor, is the current Hubble constant, is the fraction of the closure density in mass (including dark mass) and is the fraction of the closure density in dark energy. (See also: Lambda-CDM model)
The inflection point occurs at .
Rewriting the above we get:
Given that for all times since the creation of the universe, it follows that inflection occurs at:
Using current values for and , gives . Which implies that the inflection occurred when the universe was about 57% of its current size. Getting the corresponding time will require integrating the equations above with respect to time, but since the expansion was roughly linear, the 57% of size is approximately 57% of time, implying that the inflection point occurred roughly 7.8 billion years after the Big Bang, or roughly 5.9 billion years ago. Dragons flight (talk) 09:36, 18 November 2012 (UTC)[reply]
Brilliant, thanks --174.118.1.24 (talk) 17:36, 18 November 2012 (UTC)[reply]
When ΩM + ΩΛ = 1 there's an exact solution, (copied from here). That gives at the inflection point, which is about 7.1 billion years a.b.b. or 6.6 billion years ago (using parameters from here, which also give me a ≈ 0.57). -- BenRG (talk) 18:28, 18 November 2012 (UTC)[reply]

What happens if someone were to be struck by something like the Oh-My-God particle?

I mean, being hit by a 60 mph baseball is pretty painful. I mean, it was pretty unlikely for any one particular particle to have struck that particular sensor at Dugway Proving Ground, so there is likely some finite flux/second for that kind of particle. Yet there really isn't a recorded instance in history where someone suddenly suffered a severe injury (on the magnitude of a gunshot wound) suddenly and catastrophically for no reason at all. 71.207.151.227 (talk) 10:07, 18 November 2012 (UTC)[reply]

I don't think you'd notice, if as this site says, normal cosmic rays " pass through us, through our houses, through our bodies" every day, why do you think this particular form of cosmic ray would be any different? --TammyMoet (talk) 14:17, 18 November 2012 (UTC)[reply]
I think the OP was shocked by the exceptionally high energy (as I was, and the observers were). If all of that energy were absorbed by a human body, it would certainly have a significant effect on a par with a gunshot, but, fortunately for us, most of the energy is carried away by other particles without interacting significantly with the human body. Usually, damage at atomic level is not significant at cellular level, though it would be interesting to know if genetic change is caused by such particles. Perhaps they are the main driver of evolution? Dbfirs 15:35, 18 November 2012 (UTC)[reply]
Also note that 50 J is the amount of energy delivered to a typical household incandescent light bulb lit for about 1 second. That's really not that much energy from a macroscopic point of view. Dauto (talk) 16:20, 18 November 2012 (UTC)[reply]
Lesser rays are noticeable. [16] See also Cosmic ray visual phenomena. The "Oh-My-God particle" was a very rare event in the upper atmosphere and so has not been experienced yet by anyone, and until such time as it happens we can't really know what the effect would be like. Wnt (talk) 17:14, 18 November 2012 (UTC)[reply]
It could lead to a dose of a few Sievert and that can be deadly. UHECRs won't penetrate the atmosphere, hit your body and cause the equivalence of an air shower inside your body. But you can think of other ways this can theoretically happen. E.g. a cosmic ray particle can hit a dark matter particle, and that dark matter particle can then get a a similar energy. Most likely such a high energy dark matter would move through your body and the Earth, but there is then a small chance that it would interact with a nucleus in your body, giving you a potentially fatal dose of radiation. Count Iblis (talk) 17:28, 18 November 2012 (UTC)[reply]
(ec) As an aside, the detection system used (the 'Fly's Eye', a predecessor to the High Resolution Fly's Eye Cosmic Ray Detector which worked on similar principles) wasn't solely dependent on detectors on the Earth's surface, and didn't just sit and wait for a single 50-joule particle to smack into a single ground-based sensor. What the Fly's Eye picked up were showers of particles generated when a high-energy cosmic ray started to interact with the upper atmosphere; using an array of telescopes it could (literally) see these interactions by the light they produced, as collision after collision ionized atmospheric gases and generated sprays of new, fast particles that in turn triggered further ionization events. Those interactions would have occurred across several kilometers of atmosphere. To be clear, it wasn't a single *pop* and *flash* as the entire 50 joules was deposited at a single point of collision; instead, energy would be deposited along a long, branched track as the particle kicked off other fast particles produced by collisions and gradually came to rest.
In the unlikely event that such a particle did reach the Earth's surface (and a human being there) without interacting with anything else, it still wouldn't do much to a person. I don't know what the linear energy transfer rate would be for a proton in this energy regime, but I strongly suspect that the proton would emerge from the other side of your body with nearly all of its energy still intact, having deposited only a tiny, tiny faction of its 50 joules inside you. TenOfAllTrades(talk) 17:36, 18 November 2012 (UTC)[reply]
But how far does an air shower (physics) spread within a human body? Wnt (talk) 19:12, 18 November 2012 (UTC)[reply]
The density of the human body is a few thousand that of the atmosphere (at the relevant height), so you'll get a significant part of the air shower in your body (70 cm times 3000 is 2.1 km ). Count Iblis (talk) 19:48, 18 November 2012 (UTC)[reply]
The interaction length for daughter particle creation from ultrahigh energy protons is about 400 meters of air or 40 cm of water. The energy loss scale per particle is about 2-3 MeV / cm in water. If the Oh-My-God particle hit you directly, traveling vertically through you, it would have about 1.8 m to work with. So, roughly, say 3 MeV / cm * 40 cm for the initial particle + 2 * 3 MeV / cm * 40 cm for the first daughter pair + 4 * 3 MeV / cm * 40 cm for the second daughters + 8 * 3 MeV / cm * 40 cm for the third daughters + 16 * 3 MeV / cm * 20 cm for the last generation before exiting. In total, it would have deposited about 0.4 nanojoules of energy while passing through you, or roughly 0.000000001% of its energy in the 6 nanoseconds it took for the particle to enter and exit your body. You would not even notice a direct hit by such a particle. Such a particle could keep producing daughters out to the hundreds of billions. Though, even if you got him with 100 billion high energy particles, odds are you still wouldn't notice much effect. Even absorbing the full 50 J, is only an amount of heat equal to what humans produce every 0.5 seconds. 50 J is an enormous amount of energy for a single particle to have, but the energy and momentum of that particle would seem trivial to macroscopic objects like us since we are composed of roughly 7×1027 atoms. Dragons flight (talk) 22:28, 18 November 2012 (UTC)[reply]
And yet the astronauts do notice being hit with ordinary cosmic rays, even to the extent that the flickering disturbs their sleep according to the source I listed, so there must be something off there? Wnt (talk) 23:10, 18 November 2012 (UTC)[reply]
In a completely dark room, after adjusting, the eyes are sensitive to light flashes with as few as about 10 photons entering the eye. That amounts to a detection limit of about 5×10−19 J. Hence, it's not surprising that cosmic rays crossing the eyeball can be seen, but that's because the eyes are exquisitely sensitive and not because the energy is in any bulk way significant. Dragons flight (talk) 23:59, 18 November 2012 (UTC)[reply]
I remembered as a child reading about a "scorch mark" left by the passage of a cosmic ray through an astronaut's helmet, which had much impressed me, but looking into it now, and seeing your estimate of the power released, I think that this must have been a sensationalization of something like the use of parthicle track-etch technique in "the Apollo helmet dosimetry experiment". Wnt (talk) 14:58, 19 November 2012 (UTC)[reply]
If you absorb 50 J from such a process, that would roughly be equivalent to 1 Sievert which is potentially deadly. Count Iblis (talk) 01:52, 19 November 2012 (UTC)[reply]
As I pointed out, what would actually absorbed be from a direct hit by the Oh-My-God particle is negligible. You can't capture 50 J unless it has already cascaded to roughly 100 billion lower energy particles before it reaches you and yet somehow remained confined enough to hit a single person. Dragons flight (talk) 04:46, 19 November 2012 (UTC)[reply]
Yes, although the 2 daugher particles per collisions in the beginning seems a bit low i.m.o. The COM energy here is huge... Count Iblis (talk) 19:21, 19 November 2012 (UTC)[reply]



If someone did drop dead from a cosmic ray, how would the coroner be able to tell afterwards? μηδείς (talk) 04:37, 19 November 2012 (UTC)[reply]

50 J is the energy of a 60 mph baseball. That's gotta hurt! 199.111.203.215 (talk) 04:39, 19 November 2012 (UTC)[reply]

It's the momentum of grain of salt in very light breeze. You'd never notice. Dragons flight (talk) 04:46, 19 November 2012 (UTC)[reply]
That's not what the linked article says. Is there a factor of 1000 error in a calculation? Dbfirs 07:28, 19 November 2012 (UTC)[reply]
The article is talking about kinetic energy, not momentum. --Trovatore (talk) 07:47, 19 November 2012 (UTC)[reply]
Sorry Dragons flight, and thanks for the explanation, Trovatore. I must learn to read carefully before commenting! I ought to know the difference, though anyone who has been hit with a hollow-point rifle bullet will know that energy can cause a lot of damage without carrying much momentum. Dbfirs 07:39, 20 November 2012 (UTC)[reply]
  • There will be a trail of minute trace amounts of radioactive isotopes in your body. The first part of the equivalent of the air shower in your body will be due to collisions that have so much center of mass energy that they completely shatter a nucleus to pieces. The COM energy of an incoming proton of energy E = 10^20 eV and an Iron nucleus of mass m = 52 GeV at rest is sqrt[2*10^20 eV*52 GeV + (52 GeV)^2] =3.2*10^15 eV which is hundreds of times larger than the COM energy at which the LHC can carry out collisions of nuclei to create a quark gluon plasma. Count Iblis (talk) 19:18, 19 November 2012 (UTC)[reply]
So this is apparently not something that is ever likely to be observed, but may happen all the time for all we know? μηδείς (talk) 17:31, 20 November 2012 (UTC)[reply]
You would not be aware of it if it were to happen, so you would have to describe the state of the universe where you are as a superposition containing a component with a very small amplitude where such things have happened to your body. Count Iblis (talk) 17:38, 20 November 2012 (UTC)[reply]

The Philippines and total plastic bag bans (again)

I've taken the liberty of moving this to the Humanities Refdesk. That's where you should ask questions about laws. Wnt (talk) 19:10, 18 November 2012 (UTC)[reply]

Electroshocking people

If someone is holding you and you electroshock him (with a regular electroshock weapon), can the shock pass to you? And if you apply it to his arm, does it affect his legs? Comploose (talk) 16:25, 18 November 2012 (UTC)[reply]

Not sure, but you may find some of the answers (and/or sadistic pleasure) in this Brainiac clip. - Cucumber Mike (talk) 17:47, 18 November 2012 (UTC)[reply]
Of course, unless you were insulated from them by clothing, etc. Here's a pertinent demonstration with a whole bunch of people in a row and an electric fence: [[17]]. And yess, it affects the whole body, and not just the part it's applied to. There wouldn't be much point to using one if it didn't. There designed to incapacitate the target, not just make them go "Ouch!'. Dominus Vobisdu (talk) 19:20, 18 November 2012 (UTC)[reply]
No, most regular electroshock weapons have two contacts and the current flows between the two, so others in physical contact are unlikely to feel a significant shock. If there is a voltage relative to earth (as in electric fences), then the shock can be passed through many participants. Dbfirs 07:18, 19 November 2012 (UTC)[reply]

How many people are dying in Gaza?

In response to complaints, I've taken the liberty of moving this to the Humanities Refdesk. (It's not that complicated to do this if a thread bothers people) Wnt (talk) 05:14, 19 November 2012 (UTC)[reply]

Asbestos in the WTC

Was asbestos in the WTC, and was it blown in the air, or not? Comploose (talk) 18:38, 18 November 2012 (UTC)[reply]

Typing "asbestos in world trade center" into Google got me this: [18]. It looks as though it shouldn't be too hard to find the answer among those search results. --Jayron32 19:15, 18 November 2012 (UTC)[reply]
"no asbestos" in world trade center also generates many results. Comploose (talk) 20:27, 18 November 2012 (UTC)[reply]
Many of those results aren't reliable sources (conspiracytheorists etc.). Trio The Punch (talk) 22:34, 18 November 2012 (UTC)[reply]
Yep. Read this. The NYT wrote: "Anticipating a ban, the builders stopped using the materials by the time they reached the 40th floor of the north tower, the first one to go up. ". Trio The Punch (talk) 20:12, 18 November 2012 (UTC)[reply]

Lump of fat close to the heart — in roe deer in the winter season — What is it called?

I have heard that roe deer, and other wild animals living in places with cold and snowy winters, every autumn developes a big lump of fat close to the heart.
(I presume that this somehow helps them survive through the winter season).
I want to read more about this phenomenon but: what is it called? or where might be a good place to start searching for for this?
Could you please help me?89.9.197.219 (talk) 19:33, 18 November 2012 (UTC)[reply]

Quote from Bone Marrow Fat as an Indicator of Condition in Roe Deer: "Fat reserves are generally utilised sequentially starting with the subcutaneous deposits, followed by the mesenteric, kidney and finally bone marrow fat". So I think roe deer store fat in those 4 places. Trio The Punch (talk) 20:01, 18 November 2012 (UTC)[reply]
The lump of fat by the heart would be mesenteric, but there may be a specific name for it as well. μηδείς (talk) 23:10, 18 November 2012 (UTC)[reply]
The OP is possibly referring to a dewlap (accumulation of fat under the neck or at the front of and external to the ribcage), common on hooved animals - though I didn't think deer grow noticable dewlaps. The mesentery is on the other side of the diaphram and not near the heart - Medeis is wrong again. Floda 124.182.38.51 (talk) 14:49, 19 November 2012 (UTC)[reply]
It's quite possible the OP may have meant a dewlap, although the wording of the question seemed to imply something internal. And while the mesentery holding the guts in place is often referred to as "the mesentery", mesentery tissue and its derivatives occur wherever there are organs, such as the heart, situated within the coelom. For example, cardiac messentery. The OP may be referring to epicardial adipose tissue or brown adipose tissue. Without a better explanation we can't be sure. μηδείς (talk) 17:45, 19 November 2012 (UTC)[reply]
The link provided by Medeis does not provide access to the book text. I just love posters who provide links that for this and other reasons do not support their claims. Entering "cardiac mesentery" in AltaVista does not return anything that supports the idea of live mammals having a cardiac mesentery. Possibly Medeis was thinking about the mesothelium, which has a different function and is not likely to be related to the OP's question. Floda 60.228.244.240 (talk) 01:07, 20 November 2012 (UTC)[reply]
The link works for me, perhaps it's a region issue for google books? Anyway, right or wrong, Medeis seems to be linking in good faith, to an anatomy monograph from 1906. The text does list "cardiac mesentary" in a caption for a plate, saying the diagram shows "Transverse section of a tadpole, showing ... ventral cardiac mesentary." If this is not standard usage today, then perhaps the terminology has changed in the past ~100 years... SemanticMantis (talk) 02:40, 20 November 2012 (UTC)[reply]
Ahah! Tadpoles! Organisms evolutionary less developed than mammals, and embryos, are described as having a cardiac mesentery. The ventral mesentary in mammals is not cardiac as it is after the diaphram. The book access could be a regional issue or perhps is enabled if you access from within a library?? Floda 121.215.68.108 (talk) 02:50, 20 November 2012 (UTC)[reply]
Do you even know what ventral means? It's just a direction. The heart has a ventral surface as does the brain, etc. What's referred to as the ventral mesentery in humans is the mesentery closest to the belly. (Aha!) Cardiac mesentery in adult mammals doesn't disappear, it just doesn't hold the heart in place because the heart grows to fill the pericardium. It could potentially become fatty in deer, but we still don't even know what the OP is specifically talking about. Besides your bizarre need to attack me, what for I don't know, do you have anything to add to this discussion? μηδείς (talk) 03:11, 20 November 2012 (UTC)[reply]
The ventral mesentery is mesentery closest to what is the front in humans, but this is below (in humans) the diaphram as I said. What is above/in front of the diapham as a lining is the mesothelium. Any cardiac mesentery could be described as ventral, dorsal or whatever according to standard terminology, but flexibility in terminology does not mean it actually is there. Fat occurs in the vicinity of the heart but big lumps of it (to use the OP's term) would compromise breathing. So, yes, we don't know what the OP had in mind (he was quite possibly missinformed anyway - he doesn't cite a source), but fat external to the ribcage is more likely. Do you have anything usefull to add, apart from trying to defend an unsupported opinion by personal attack? Your link doesn't count as according to SemanticMantis it is about tadpoles. You might as well claim that deer do not have diaphrams and breath via gills. Floda 121.215.68.108 (talk) 04:01, 20 November 2012 (UTC)[reply]

@ Semantic Mantis, I simply provided a copy to the first link at google that mentioned cardiac mesentery; it wasn't really meant to be relevant to the deer question as such, just to show that mesentery is a type of tissue found in relation to organs that float in the coelom, not some sort of organ that resides only in the abdomen. This article is interesting, it mentions studies of fatty female elk hearts themselves, as well as fat surrounding them on the pericardium, as well as the kidneys. They study this muscular and mesenteric fat because it is a sign of fertility. I presume they don't study bone marrow fat since it would only be lost in starving deer and subcutaneous fat since it would be a sign of obesity. Searches for "roe deer dewlap" don't seem to get any relevant hits and image searches aren't encouraging. American elk don't seem to have dewlaps either. Moose do, but they hardly seem to be fat storage organs. μηδείς (talk) 03:52, 20 November 2012 (UTC)[reply]

Again, Medeis, you have provided a link that does not support your claim. Yes, fat occurs around the heart, but the link does not even use the term mesentery, and does not in any way indicate "big lumps". Floda 121.215.68.108 (talk) 04:01, 20 November 2012 (UTC)[reply]
  • From Trio The Punch we know that mesenteric fat is important in the roe deer, and Medeis referenced epicardial adipose tissue. In better-studied organisms (humans) we know that there are indeed epicardial fat deposits which share a common embryological origin with those of the mesentery.[19][20] While researchers show more interest in the pathology, this fat serves as a source of triglycerides and perhaps regulatory signals affecting the heart and arteries in subtle ways (the object of their present research). "Epicardial fat can be abundant especially in ruminants" and is absent in starving animals, and the pointed shape of the apex of the heart "can be pronounced in starving animals due to lack of epicardial fat." [21] At this point we're probably nearing the edge of what is known; roe deer are not model organisms and the amount of effort people have put in to figure out the role of epicardial fat is probably limited. I doubt that anyone has really liposuctioned it from the animals and seen what happens to their cardiovascular fitness or whatever. But you can do a lot of reading about epicardial fat in other organisms.[22] Wnt (talk) 18:06, 20 November 2012 (UTC)[reply]

Floda and Medeis, would you please both be so kind to refrain from interacting with/talking about the other person? This message is carefully worded to be as neutral as possible, but that does not mean I don't have an opinion. According to the rules of the internet you both lost the debate. Jimbo won, because he did not participate. This is not an invitation to say something like: "I'll stop talking to x because x is an idiot". There is no need to respond to this message. Thanks in advance, Trio The Punch (talk) 20:40, 20 November 2012 (UTC)[reply]

November 19

manual lenses in bridge cameras

Besides Fujifilm what others brands make super-zooms cameras with a lens capable of doing manual zoom and manual focus??
recently a friend was looking for a super-zoom, and showed me a canon SX – something, and it felt like a toy in my hands, and he don’t like the Fuji’s IQ,
are any Sony, Canon, or Nikon camera bridge (with a zoom around 18~30x) like the hs line of fuji??

Iskánder Vigoa Pérez (talk) 00:24, 19 November 2012 (UTC)[reply]

From bridge camera "With zoom ranges and sales rapidly increasing in the early 21st century, every major camera manufacturer has at least one 'super zoom' in their lineup." So, just wait and see. Or try one like the Sony NEX-some # or one of its concurrents. [23] is a good source of such information. Philoknow (talk) 16:35, 19 November 2012 (UTC)[reply]
Yes, sure, but talking about now?? There isn’t any 15-30x camera capable of manual zoom and focus… or at least manual focus?
the canon sx40 looks just like a toy wat side of my mom’s fuji hs20
my friend has family that will come to Cuba next week and he wants to exploit this opportunity
Iskánder Vigoa Pérez (talk) 19:39, 19 November 2012 (UTC)[reply]
It may help if you further explain your question. I assume many bridge cameras probably even most bridge cameras have manual focusing at lead thats what our article and many other discussions seem to suggest. Heck I think even some higher end P&S or those at the crossover point have manual focusing (I seem to recall using at least camera with it and a search also finds mention of manual focus in some P&S cameras) although it may be very hard to use due the difficult focusing on the LCD (our article mentions many bridge cameras provide a zoomed in focus square), it's ultimately primarily a software issue. But your original question makes me think you want more then simple manual focusing. Do you want manual focusing using some sort of focus ring or similar rather then adjusting a slider on screen with cursors? (Your second question also seems to reaffirm this point. Surely nearly even single non fixed zoom camera offers some sort of manual zoom even the vast majority of P&S ones. i guess some may have some sort of point to zoom or try to zoom in faces when set to potrait mode but manually zooming must be the way most people use zoom cameras. Even cameras without a zooming lens will often provide zooing controls due to 'digital zoom'. Of course the controls will often be a simple cursor or with modern touch screen controls perhaps an on screen slider.)
In any case a simple search for 'bridge camera manual focus' finds this review [24] which discussess manual focus methods in various recent bridge cameras most of which appear to have fairly high zooming levels. There are also lots of other older discussions from the same search, while many of the models may now be old you should be able to find their replacements and see what they do. Of course if you can't find anything fulfilling your requirements your best bet may be to consider a DSLR, its not clear why your focused on bridge cameras. Pricewise it's often not clear cut nowadays so I assume its a size issue but if you have special requirements you may have no choice.
Nil Einne (talk) 01:44, 20 November 2012 (UTC)[reply]
Yes, I meant a bridge camera with two rings, a focus ring and a zoom ring… (just like the Fujifilm’s HS)I recommended him to buy a d3200 and a cheap tamron18-200 (he can afford it) but he wants something larger, 20x or more…
I guess the IQ of a picture taken whit this camera/lens, at 200mm and cropped to the half will still be better than the IQ of a picture of any bridge… and making math that’s 22x or so, I told him but he wants a bridge camera with 20 x or more…
Do you know any model available right now (from canon, nikon, sony or pentax), or are the fujis the only choice?
Thanks for the answer!
Iskánder Vigoa Pérez (talk) 03:19, 20 November 2012 (UTC)[reply]

pneumonia survival rates

what was the pneumonia survival rate before antibiotics, and what is it now?--Wrk678 (talk) 01:40, 19 November 2012 (UTC)[reply]

From the article Pnemonia#Management: 10% mortality now vs 50% before antibiotics for patients who end up in the hospital. RudolfRed (talk) 02:47, 19 November 2012 (UTC)[reply]

yes but not everyone ended up is hospital back then either, so I need rates for those that died at home.--Wrk678 (talk) 03:31, 19 November 2012 (UTC)[reply]

I found this page: http://www.jci.org/articles/view/29920, which says 30-40% fatality rate. RudolfRed (talk) 04:28, 19 November 2012 (UTC)[reply]
Also note that the fatality rate may be different for different strains. StuRat (talk) 03:43, 20 November 2012 (UTC)[reply]

Element transmutation

Can iron transmute to silver?--Almuhammedi (talk) 13:40, 19 November 2012 (UTC)[reply]

Probably not. Iron is at the apex of the nuclear binding energy graph, which means that it is the hardest element to transmute out of. To put it another way, given a long enough time frame, thermodynamically the entire universe should be transmuting into iron. Nuclear fusion of smaller elements in stars produces iron as the ultimate end product, and spontaneous fission shouldn't produce any element smaller than iron. --Jayron32 13:47, 19 November 2012 (UTC)[reply]
Well sure, but that's not really the question being asked. (Strictly speaking, because of activation energy barriers associated with nuclear fusion, what you would actually get if you wait long enough is a cold, near-perfect vacuum contaminated ever-so-slightly with hydrogen, helium, and an elemental assortment enriched in stuff around the iron peak. Further, most grand unified theories predict that all 'stable' nuclei and even bare protons will eventually decay, with a half life on the rough order of 1036 years. See, for instance, proton decay, ultimate fate of the universe, and future of an expanding universe.)
Energetically-unfavorable transmutation is certainly possible, and happens on a regular basis in laboratories all over the world. It also occurs in nature; on the largest scale supernova nucleosynthesis is responsible for virtually all of the heavier-than-iron elements in the universe. (One could argue on that basis, not entirely speciously, that much of the silver we have already is the result of transmutation of iron.) Whether the multiple steps required to get from iron (atomic weight around 56) up to silver (atomic weight around 107) could be carried out industrially with meaningful yields is questionable, and whether it could be done economically is doubtful in the extreme. TenOfAllTrades(talk) 15:49, 19 November 2012 (UTC)[reply]
This does not answer your question, but it is Quite Interesting. See also Synthesis of precious metals. Trio The Punch (talk) 13:53, 19 November 2012 (UTC)[reply]
It used to be that a nuclear engineer learned how to read the periodic table, suitably modified for radiochemistry. Here are some educational decay charts from Lawrence Berkeley National Lab. I know of no direct transition from iron to silver: but you can sum over the known probabilities of all possible intermediate transition probabilities and determine exactly how unlikely the transition is. Like many processes in nuclear physics, the probability of iron-to-silver transmutation is probably tiny but non-zero. For perspective, consider how much silver we have on earth, and how much hydrogen we started with in the original phase of stellar nucleosynthesis. That ought to give you a sort of intuitive idea about the scale of the probability. Start with a sun-sized blob of hydrogen, and after a few billion years, you end up with a tiny amount - maybe a few thousand tons - of silver. The probability of hydrogen transmuting to silver is very small, and yet, on a cosmological scale, even very rare events can occur. Nimur (talk) 13:59, 19 November 2012 (UTC)[reply]
Given you have a nice neutron source like a nuclear reactor you will transmute 1g of iron into a few mg of silver and a lot of other elements over time (most likely years). The whole thing will be radioactive like hell but if you wait 10 half-lifes you are save.--Stone (talk) 14:19, 19 November 2012 (UTC)[reply]
  • My feeling is that most silver has originated by transmutation from iron (supernova nucleosynthesis) - but it turns out your workbench has to include the right kind of supernova, an "r-process-rich star". A second r process, to be precise. This is apparently explained in [25], but that is not an easy read! Found via [26] which is far too simple. Still... if you can do it in a supernova, then this is just a matter of engineering. :) Wnt (talk) 17:00, 19 November 2012 (UTC)[reply]

It would be more economically feasible to get to Ag from Cd, by irradiating stable 106Cd with neutrons (producing 107Cd, which undergoes electron capture to 107mAg, which decays by IT to stable 107Ag). Double sharp (talk) 06:22, 20 November 2012 (UTC)[reply]

Technically, 106Cd is only observationally stable, and can theoretically decay by β+β+ to 106Pd with a half-life of over 4.1×1020 a, but that doesn't make any difference as the half-life is so long that the decay has never actually been observed. Double sharp (talk) 06:26, 20 November 2012 (UTC)[reply]

Is this a fallacy? Classifying people from a psychological perspective

If you classify people, mothers for example, into angry, controlling, narcissistic, envious, emotionally unavailable mothers, aren't you forcing reality into your categories, ignoring that anyone, any person, can have one of the typical behavior of each and every category above? Wouldn't that be a series of categories without typical specimen, since you'll end up pushing mothers into the category that better fits yours feelings? Philoknow (talk) 16:52, 19 November 2012 (UTC)[reply]

Hmmm... now there's an interesting psych experiment. Given well trained actors with detailed and well-selected instructions, who assemble an artificial research community (e.g. a supposed Mars mission psych test) can you induce any individual to fall into one of these types regardless of their previous personality? Wnt (talk) 17:03, 19 November 2012 (UTC)[reply]
Is that an allusion to a thought experiment that really happened or just the creation of one? Philoknow (talk) 17:04, 19 November 2012 (UTC)[reply]
Just me piggybacking on your question, sorry. Wnt (talk) 19:15, 19 November 2012 (UTC)[reply]
The mother issue is just an example I came across. But there are alternative categories for co-workers, partners, and whatever you want. How to you call this kind of errors in thinking? If you have too little evidence for them, isn't there a too-little-evidence-for-the conclusion fallacy? Philoknow (talk) 17:14, 19 November 2012 (UTC)[reply]
It would be the drawing of conclusions from application of invalid categories that might be fallacious, again, see the list and look for one that seems to cover what you are thinking of. (That's just a matter of what fallacy means--not all mistakes in reasoning are fallacies.) Perhaps you could give a more detailed example of the sort of thing you have in mind. Perhaps you mean someone who forms an immediate judgment of a person as a type of "x" and then interprets all that persons actions as being due to their being an "x" regardless of evidence? That would be stereotyping, which literally means solid (as in the sense of unchangeable) categorizing. μηδείς (talk) 17:58, 19 November 2012 (UTC)[reply]
I'd say it's a misconception, but not an error in the reasoning. For more on this check Scientific misconceptions and list of common misconceptions. OsmanRF34 (talk) 18:10, 19 November 2012 (UTC)[reply]
What is this fallacy?: Report says that college students who talk to parents at least once a week are less likely to have trouble with abusing alchohol/drugs, therefore parents should call child at least once a week. Logically I would say that the student who talks to parents once a week is already more responsible, etc. which is why they call/ talk to parents regularly.165.212.189.187 (talk) 19:11, 19 November 2012 (UTC)[reply]
To me, this seems like making a correlation equal to causation. What the name for this fallacy is, I don't know, but correlation=/= causation.128.227.105.163 (talk) 20:14, 19 November 2012 (UTC)[reply]
It would be correlation does not imply causation. --140.180.252.13 (talk) 20:14, 19 November 2012 (UTC)[reply]

The following is what Carl Jung wrote in a letter to Sigmund Freud dated 12th June 1911 : "My evenings are taken up largely with astrology. I make up horoscopic calculations in oder to find a clue to the core of psychological truth. Some remarkable things have turned up which will certainly appear incredible to you . . . . . . I dare say we will one day discover in astrology a good deal of knowledge that has been intuitively projected into the heavens". Cinquefoil (talk) 19:08, 19 November 2012 (UTC)[reply]

If we didn't "force reality into our own categories", there'd be no way to reason or talk about anything (try it.) Psychologizing as a form of argument, however (speculating about the motives/psychological causes etc as to why one's opponent hold the views they hold, and attacking the former instead of the latter) is a bad thing Asmrulz (talk) 09:17, 20 November 2012 (UTC)[reply]

Impact depth

This article gives the formula for the impact depth as derived by Newton. How can it be derived? The article gives a sketch but doesn't have the details. 65.92.7.202 (talk) 16:59, 19 November 2012 (UTC)[reply]

The key criteria from it are "impactor carries a given momentum. To stop the impactor, this momentum must be transferred [...] to the material (mass) directly in front of the impactor, which will be pushed at the impactor's speed. If the impactor has pushed a mass equal to its own mass at this speed, its whole momentum has been transferred to the mass in front of it and the impactor will be stopped." Write the equation for the momentum of the projectile; substitute a function of the the projectile's density and length for the mass term. Do the same for the target material. Set them equal to each other (conservation of momentum) and solve for...whatever. DMacks (talk) 18:39, 19 November 2012 (UTC)[reply]

Iron Dome

Is the Patriot missile much worse or expensive than the Iron dome? If the Israelis find the rockets from the Palestinians so disturbing, why didn't they have lots of Iron Domes? According to the article they are not prohibitively expensive. If you compare them to a land incursion they are quite cheap. Philoknow (talk) 17:24, 19 November 2012 (UTC)[reply]

It's a different job. Patriot defends against long range missiles which descend on the target at very high speeds. Iron dome defends against short range threats, which approach at relatively low speeds. Israel's equivalent of Patriot is Arrow (Israeli missile). -- Finlay McWalterTalk 17:40, 19 November 2012 (UTC)[reply]
According to the article, some Israelis do think Iron Dome systems are too expensive. --Demiurge1000 (talk) 17:51, 19 November 2012 (UTC)[reply]
"too expensive" comparing to a Palestinian Quassam rocket. OsmanRF34 (talk) 18:13, 19 November 2012 (UTC)[reply]
Iron Dome is a new system. It still has some problems and is still in development. In the future they will likely have many more of them. Ruslik_Zero 18:54, 19 November 2012 (UTC)[reply]
It already successfully intercepts at least 70% of the rockets launched against populated areas (as much as 90% according to the Jerusalem Post). 24.23.196.85 (talk) 07:01, 20 November 2012 (UTC)[reply]

Universal milling machine or turret milling machine....

Hi I want to know which type of milling machine has better accuracy universal or turret milling machine. — Preceding unsigned comment added by 117.219.148.78 (talk) 19:31, 19 November 2012 (UTC)[reply]

A turret mill is generally less rigid. However, how accurately you machine any workpiece is dependent on so many factors I would not like to make a claim either way. If the job is done with the correct approach, the machines are adequately looked after, correct speeds and tools used, there is no reason why any properly designed machine tool should not be used at an accuracy determined by the machinist's available measuring instruments. Wickwack 120.145.142.95 (talk) 01:29, 20 November 2012 (UTC)[reply]

November 20

What's the solution to Haldane's dilemma?

Haldane's dilemma (edit | talk | history | protect | delete | links | watch | logs | views)

I read the article and still don't understand. Sagittarian Milky Way (talk) 03:23, 20 November 2012 (UTC)[reply]

The article doesn't seem to provide a great or simple overview of the topic, but from what I understand, the "dilemma" is due to some people's interpretation that a mathematical model can be used to make a vague and nonquantitative prediction in contradiction to observations. It seems to me that the "resolution" is not to trust the flawed mathematical formulation, because it makes vaguely incorrect predictions. Where are you getting stuck? Lots of scientists have designed models, or created mathematical equations, that turned out to be wrong. Even more scientists have produced proper, accurate models, that are misunderstood and misused by others to make erroneous claims. Perhaps you'd do better to ignore the "dilemma" and simply read our better article about population genetics.Nimur (talk) 05:23, 20 November 2012 (UTC)[reply]
I'll read that, tomorrow. Interesting I'm sure. Sagittarian Milky Way (talk) 07:05, 20 November 2012 (UTC)[reply]
Yup: from my understanding of the article, the 'solution' is that the dilemma never existed in the first place: Haldane was making assumptions that don't accord with real genetic/evolutionary processes, so his conclusions were invalid. AndyTheGrump (talk) 05:31, 20 November 2012 (UTC)[reply]
  • See [27]. The number of mutational changes that occur really is much greater than the number that become fixed - most of the time, random mutations are neutral mutations. There is also the truncation selection, which I would say is relevant in so-called "junk DNA" - a concept which is now growing old, but which at least accurately reflects the point that regulatory sequences are prone to constant fine-tuning and there are a bazillion different ways to get the same effect. (You can have a 6 base pair motif that isn't exact anywhere in a fairly loose region of DNA and the outcome may work out to be the same pattern of regulation) So it doesn't really matter how evolution serves you up a "strong COMT promoter" or a "weak COMT promoter", depending on whether warriors or worriers are doing better against the hyenas this season. There's no need for fixation of one precise change because there are so many ways to strengthen or weaken the binding of a particular protein to the promoter. Wnt (talk) 05:35, 20 November 2012 (UTC)[reply]
Well then the article should say that then. It's actually very simple. You're left with the impression that a resolution hasn't been found in 55 years despite extremely embiggened knowledge of genetics and the fact that scientists like explaining things. Besides "we know which side is correct" (the fossil record/radioisotopic dating/genetics and all that) (which is not mentioned by the way).
Hey, are there any other paradoxes which we know/knew which side is correct but don't know why? Sagittarian Milky Way (talk) 07:01, 20 November 2012 (UTC)[reply]
The article clearly needs work but it seems to accurately report that the dilemma is no real dilemma. For example right in the LEDE it says:
Contrary to creationist claims, Haldane's dilemma is of no importance in the evolutionary genetics literature
....
Haldane stated at the time of publication "I am quite aware that my conclusions will probably need drastic revision", and subsequent corrected calculations found that the cost disappears. He had made an invalid simplifying assumption which negated his assumption of constant population size, and had also incorrectly assumed that two mutations would take twice as long to reach fixation as one, while sexual recombination means that two can be selected simultaneously so that both reach fixation more quickly. The creationist claim is based on further errors and invalid assumptions
Nil Einne (talk) 14:35, 20 November 2012 (UTC)[reply]
As far as I can see just having had a quick look there is a basic flaw in that in reality the main competitors for an individual in a species are other members of that species. They do not normally compete with nature, they compete with each other. Thus his rate of death does not matter much normally when the environment is only changing slowly. Of course all bets are off nowadays that humans are around and killing everything in sight but catastrophes like that aren't the norm. Dmcq (talk) 10:12, 20 November 2012 (UTC)[reply]

Bear pig

What animal is equally related to a bear and a pig?GeeBIGS (talk) 05:47, 20 November 2012 (UTC)[reply]

What do you mean "Equally related"? I'm not sure I follow your question... --Jayron32 05:53, 20 November 2012 (UTC)[reply]

Did they have a common ancestor? What did it look like? Similar to anything extant today?GeeBIGS (talk) 05:56, 20 November 2012 (UTC)[reply]

Ah. It would have to be a pretty old ancestor. Taxonomically speaking, the smallest grouping that both bears and pigs share is the clade Laurasiatheria. I'm not entirely sure what the nearest common ancestor for that clade is, but it's got to be a long time ago, among mammals. --Jayron32 06:08, 20 November 2012 (UTC)[reply]

What would a hybrid possibly look like?GeeBIGS (talk) 06:00, 20 November 2012 (UTC)[reply]

I don't know that bears and pigs are genetically close enough to hybridize. Most hybrids need to share a genus, usually. Bears and pigs don't even occupy the same order (Carnivora vs. Artiodactyla). --Jayron32 06:08, 20 November 2012 (UTC)[reply]
You can answer many questions of this type pretty easily on Wikipedia (I haven't looked for any recent updates on NCBI). Look up bear. Look up pig. Look up class, order, etc in the table at right. Both are mammals; the bear is order Carnivora, the pig is order Artiodactyla. Now look those up and you'll see both of these are in superorder Laurasiatheria. Now - look for anything in Laurasiathera that is not in Carnivora or Artiodactyla (well, Cetartiodactyla and Ferae, once you look at the tree) - that gets you Perissodactyla, any odd-toed ungulate, as a potential answer, though as the branch is unresolved in the taxonomy shown it's still possible it's closer to one or the other. Oh, and yes -- the way you phrase the question, anything more distantly related also qualifies, plants, yeasts, sea urchins etc. But looking for the closest animal equally related to both is the more interesting question. :) Wnt (talk) 06:09, 20 November 2012 (UTC)[reply]
I am satisfied with a tapir for now.GeeBIGS (talk) 06:20, 20 November 2012 (UTC)[reply]

Depending on who asked the question, the answer they were looking for may have been Al Gore. But taxonomically, Jayron is right. However the earliest Laurasiathere certainly did not look like a Tapir, and probably looked more like a rat or an opossum. μηδείς (talk) 17:28, 20 November 2012 (UTC)[reply]

GeeBIGS—you ask the question, "What would a hybrid possibly look like?" Is the question what actual animal a possible hybrid would look like? Or is the question what nonexistent animal a "hybrid" would look like? I believe there is software that makes an image metamorphose into another image. It does so over many steps. One of the middle steps would be a credible transitional animal between bear and pig. Bus stop (talk) 17:39, 20 November 2012 (UTC)[reply]
An attempt at this research was reported 2008-4-1.[28] Of course, this is discredited; it would be unethical to use cells from a bear.[29] Wnt (talk) 18:18, 20 November 2012 (UTC)[reply]
Earlier, unconfirmed reports had existed for almost two years at that point. --Jayron32 19:32, 20 November 2012 (UTC)[reply]

How can I separate mixture of water and ammonia?

Not sure if boiling or freezing works--124.172.170.234 (talk) 06:39, 20 November 2012 (UTC)[reply]

Probably neither directly; ammonia as a gas is hard to seperate out by distillation. Instead, acidify the solution careful to the equivalence point, with (for example) hydrochloric acid. This should allow you to distill the water off from the resultant ammonium chloride. --Jayron32 06:50, 20 November 2012 (UTC)[reply]
If it's a reasonably concentrated solution, there's a pretty high vapor pressure of ammonia over the solution especially when warmed (see ammonium hydroxide, and in general you can boil some ammonia out (see ammonia#Properties. But the answer really depends really what the questioner's goal is (actually "separate the two components" vs "get some pure water from it" vs "get some pure ammonia from it"). Jayron's got the right start for the old industrial process (see ammonia#Synthesis and production). DMacks (talk) 08:36, 20 November 2012 (UTC)[reply]

9th period

Are there any relativistic predictions for the 9th period of the PT (Z > 172), just as there are for the 8th (Pyykkö's)? (Note that Z = 173 is not actually predicted to be an end of the PT anymore – see Periodic table.)

Double sharp (talk) 07:16, 20 November 2012 (UTC)[reply]

How much more diverse would the species be if we hadn't gone through the Toba catastrophe?

Some alleles were lost forever, right? But I mean, how much more freaking diverse do you want the human race to be? Would we start having elf ears or white hair at youth or tribes with very strong bonds without ritual circumscision that only like making love? Would the rarer traits like Zulu veins or Sarah Baartman's anterior be more common? Perhaps some more blood types or major histocompatibility complexes or body odors? And about how close are we to filling the a full subspecies of genetic diversity? Not much, maybe? How much of the Homo sapiens genetic field is extinct? Sagittarian Milky Way (talk) 07:44, 20 November 2012 (UTC)[reply]

I am not sure which I find more troubling, your fear of elves or of people who like sex. μηδείς (talk) 17:17, 20 November 2012 (UTC)[reply]
(ec) As a first point, the idea that the Toba supereruption caused a genetic bottleneck doesn't get much support nowadays -- there was very likely a major bottleneck, but much earlier than that. In any case, genetic diversity does not necessarily show up as overt physical differences. For example, pygmies in West Africa show a considerably higher level of genetic diversity than a group of randomly chosen Europeans. Generally, though, if you want to see the effects of diversity, look at sub-Saharan Africa -- that's where diversity is greatest. Looie496 (talk) 17:19, 20 November 2012 (UTC)[reply]
The article on Toba doesn't seem as deinitive as you are. Rmhermen (talk) 19:11, 20 November 2012 (UTC)[reply]
I didn't think I was being all that definitive, but, well, the article hasn't really kept up with events. The Toba eruption is pretty clearly dated to about 70000 years ago. It has recently become clear that the rate of human genetic change is about half as high as the value that was used for a long time (PMID 22965354), and when you revise dates accordingly, the numbers don't even come close to working out. Looie496 (talk) 19:32, 20 November 2012 (UTC)[reply]
Stop talking about my anterior please. Sarah Baartman (talk) 18:02, 20 November 2012 (UTC)[reply]

free electrons per atom

How do you calculate ab initio the number of free electrons per atom. For example, Tin is found in both the +2 and +4 oxidation states. Which ones do you use for a resistivity or a mean free path calculation? However, for graphite, based on what I know about organic chemistry, it would be 6 pi electrons per six carbons, or 1. (Yet carbon is frequently found in a +4 oxidation state, like carbon dioxide). Which oxidation states do you use in the context of electrons being on the scale of the fermi velocity? 71.207.151.227 (talk) 11:04, 20 November 2012 (UTC)[reply]

Help! This is urgent. Why is it so hard to find such a list of such simple values on the internet? 71.207.151.227 (talk) 11:12, 20 November 2012 (UTC)[reply]

A better question might be, why do you think oxidation numbers have anything to do with the number of free electrons in a solid? The Fermi energy, by itself, tells you the effective density of free electrons. See for example this discussion [30]. There is also a table of Fermi energies / velocities [31]. Dragons flight (talk) 11:27, 20 November 2012 (UTC)[reply]
I'm supposed to calculate the Fermi velocities ab initio, so that won't do. 71.207.151.227 (talk) 11:57, 20 November 2012 (UTC)[reply]
You can calculate Fermi energy / free electron density from the resistivity, if you know that. What are you allowed to use, cause oxidation states probably aren't going to be what you want. Dragons flight (talk) 12:06, 20 November 2012 (UTC)[reply]

heat capacity of a plasma

What's the model for the heat capacity of a plasma? Is C_P still C_V + R? Are there lattice vibrations in a plasma? 71.207.151.227 (talk) 11:55, 20 November 2012 (UTC)[reply]

A plasma is generally approximated as a gas of partially or fully ionized nuclei superimposed with a gas of free electrons. There is no lattice in a plasma. Dragons flight (talk) 12:29, 20 November 2012 (UTC)[reply]

Isolated systems

Do true thermodynamically isolated systems occur in biology? If not, what about anywhere in the natural world? Are even closed systems possible in biology? I've heard an egg could be. — Preceding unsigned comment added by 94.116.147.229 (talk) 15:56, 20 November 2012 (UTC)[reply]

No, entirely isolated systems do not exist, they are a theoretical concept that can however be used as an approximation of a naturally occurring system in some cases. The only exception is the universe itself, because it by definition contains all matter and energy in existence, so that excludes the possibility of external interaction. And yes, an egg for all practical purposes can be considered a closed system, though I think that very small particles like hydrogen molecules could leak through the shell. - Lindert (talk) 16:13, 20 November 2012 (UTC)[reply]
Read Thermodynamic system#Isolated_system and Isolated system. Sarah Baartman (talk) 16:38, 20 November 2012 (UTC)[reply]
If an egg was a closed system, you wouldn't be able to hard-boil it! Remember, a closed system doesn't just prevent transfer of matter, but of heat/energy as well. 209.131.76.183 (talk) 18:37, 20 November 2012 (UTC)[reply]
No, that's an isolated system. A closed system can exchange energy but not matter. --Jayron32 19:30, 20 November 2012 (UTC)[reply]

Fuel consumption of engines

For small engines the fuel consumption is measured in g/kWh. If we multiple the SFC with the energy density of the fuel (44.4MJ/kg for gasoline in this case) we get the following expression: 300 g/kWh * 44400MJ/g = 13MJ/3.6MJ. Since the units cancel out we get a ratio, and it appears to be the inverse of the efficiency of the engine. My question is: is this actually the inverse of the efficiency? I double checked my work and everything seems correct but I'm having a hard time convincing myself that calculating the efficiency is as simple as a single multiplication.A8875 (talk) 16:29, 20 November 2012 (UTC)[reply]

Momentum transfers

I have absolutely no clue how to solve this type of problem. How do I figure out velocities after a collision or before it from the equation ? This in in one dimension. --Melab±1 16:44, 20 November 2012 (UTC)[reply]

Conservation of energy, or if it isn't conserved you need to specify an additional condition (e.g if the masses stick to each other after the collision that imples a relation between the velocities after the collision). Count Iblis (talk) 17:13, 20 November 2012 (UTC)[reply]
It is a totally elastic collision and the masses are not necessarily equal. I'm looking for the general solution. I tried solving for but that did not work. --Melab±1 17:18, 20 November 2012 (UTC)[reply]
m1 u1 + m2 u2 = m1 v1 + m2 v2
Totally inelastic collision: v1 = v2, so we have:
v1 = (m1 u1 + m2 u2)/(m1 + m2)
Count Iblis (talk) 17:25, 20 November 2012 (UTC)[reply]
But it can't be known for sure that or that . It could be that or or both. --Melab±1 17:34, 20 November 2012 (UTC)[reply]
If the collision is totally inelastic, then the velocities after the collision are the same. Count Iblis (talk) 17:42, 20 November 2012 (UTC)[reply]
You can't solve for two variables if you only have one equation. This problem can't be solved without more information. Looie496 (talk) 17:44, 20 November 2012 (UTC)[reply]
Well, the problem I am working with only gives me the masses and their velocities prior to colliding. It is an elastic collision. They don't stick together. --Melab±1 17:45, 20 November 2012 (UTC)[reply]
Have a look at Elastic_collision#One-dimensional_Newtonian. There are worked examples. You have TWO equations, one for conservation of momentum (which you wrote), and another for conservation of energy (which you did not). Using both, you can solve for new velocities, regardless of how the initial masses and velocities compare. The algebra can get rough, but our article also describes a nice trick of changing the frame of reference, so that one of the initial masses has zero velocity. SemanticMantis (talk) 17:56, 20 November 2012 (UTC) P.S. I have no Idea what Iblis is thinking above. Of course the velocities can be different after collision... [reply]
It did say inelastic at one point. I guess my edit got through after his. So does the and on that page equal and , respectively? Even still, I can't understand it. --Melab±1 18:05, 20 November 2012 (UTC)[reply]


Apologies, I misread "elastic" for "inelastic". In that case, you have conservation of energy, but it is not recommended to use the equation for energy conservation, because the algebra then gets messy. Instead, you work in the center of mass frame where the total momentum is zero. Suppose we are in the center of mass frame, then the energy of two particles with momenta p1 and p2 with masses m1 and m2 is given by:

E = p1^2/m1 + p2^2/m2 = (p1^2) (1/m1 + 1/m2)

because in the center of mass p1 = -p2. Energy conservation thus implies that the magnitude of the momentum of the partices is the same before and after the collision.

The velocity of the center of mass is

u* = (m u1 + mu2 )/(m1+m2)

The initial velocities in the center of mass system are thus:

u1' = u1 - u*

u2' = u2 - u*

The two momenta in the center of mass frame are

p1 = m1 u1'

and

p2 = m2 u2'

they are equal in magnitude and they add to zero, so p1 = -p2

This means that after the collision, the momenta reverse sign, so particle 2 will get the momentum that particle 1 had and vice versa. So, we have:

m1 v1' = p2 = m2 u2' = m2 (u2-u*) ---->

v1' = m2 (u2-u*)/m1

The velocity of particle 1 after the collision in the original frame is thus:

v1 - u* = m2 (u2-u*)/m1 ------>

v1 = u* + m2 (u2-u*)/m1

Count Iblis (talk) 18:13, 20 November 2012 (UTC)[reply]

Or, if you want to avoid using the center of mass solution, you can use the energy conservation (not as messy as Count Iblis implied). Start with the energy conservation

m1 u1^2 + m2 u2^2 = m1 v1^2 + m2 v2^2

and rearrange the terms as

m1 (u1^2 - v1^2) = m2 (v2^2 - u2^2)

Open each parenthesis as a product of two factors

m1 (u1 - v1)(u1 + v1) = m2 (v2 - u2)(v2 + u2) (Equation 1)

use the momentum equation

m1 u1 + m2 u2 = m1 v1 + m2 v2

rearranged as

m1 (u1 - v1) = m2 (v2 - u2) (Equation 2)

to substitute the factors m1 (u1 - v1) into the equation 1 above and get

m2 (v2 - u2)(u1 + v1) = m2 (v2 - u2)(v2 + u2)

which simplifies to

u1 + v1 = u2 + v2 (Equation 3)

Equations 2 and 3 above now form a set of linear equations that can be easily solved.

Dauto (talk) 19:00, 20 November 2012 (UTC)[reply]

Recycling 10, 20, or 30 year old paper

Hello, I have some old documents in boxes, and these papers are probably 10, 20, or 30 years old. I intend to recycle all of these documents, and I was wondering, can they even reuse the fibers from paper that old? 2A02:AF8:1:3500:0:0:0:9868 (talk) 21:30, 20 November 2012 (UTC)[reply]